Pediatric Questions With Answers

April 22, 2017 | Author: clinicalskills | Category: N/A
Share Embed Donate


Short Description

Download Pediatric Questions With Answers...

Description

Assessment Exam for USMLE Step 2: Pediatrics *

Answers and Explanations

*USMLE is a joint program of the Federation of State Medical Boards of the United States, Inc. and the National Board of Medical Examiners.

©2003 Kaplan, Inc. All rights reserved. No part of this book may be reproduced in any form, by photostat, microfilm, xerography or any other means, or incorporated into any information retrieval system, electronic or mechanical, without the written permission of Kaplan, Inc. Not for resale.

Pediatrics Assessment Exam 1.

During an annual visit, a 5-year-old boy is noted to have a heart murmur. The murmur is soft systolic, grade II in intensity, best heard along the left lower sternal border, and does not radiate. The murmur decreases in intensity in the sitting position. The first and second heart sounds are normal and no gallop is heard. The patient is an active child with no symptoms of tiredness or breathlessness, and he has a good appetite. He was hospitalized as an infant for bronchiolitis. There is no other history of lung infections. His weight and height parameters are at the fiftieth percentile. Lung examination reveals good breath sounds and no rales. There is no hepatomegaly or peripheral edema. The parents are concerned when they are told about the murmur. Which of the following is the most appropriate next step in management? (A) (B) (C) (D) (E)

Obtain a chest x-ray Obtain a hematocrit Perform an electrocardiogram Reassure the parents that the child is healthy Send the child for an echocardiogram

2.

A 7-year-old boy is brought to the physician because of a new-onset rash on the trunk that appeared after a recent family camping trip in central Pennsylvania. The mother remembers that her son had a short-lived “flu” for several days on returning from camping that resolved with over-the-counter cold medication. Approximately 2 weeks later they noticed a red spot develop on his flank that has since been enlarging. They tried applying cortisone cream to it with no result. The boy is in good overall health and has no past history of serious medical problems. When the physician inquires about possible insect bites during the trip, the mother reports that she had removed several ticks from her son during their trip. No other family member has had similar symptoms and the family history is unremarkable. On physical examination the patient is in no acute distress. His vital signs are within normal limits. Inspection of the skin reveals a large, annular, erythematous plaque with central clearing present on the right flank. The largest diameter of the skin lesion is approximately 12 cm. Axillary and inguinal lymph nodes are not palpable. Potassium hydroxide preparation of skin scrapings from the edge of the lesion is negative. Which of the following is the most appropriate treatment for this patient? (A) (B) (C) (D) (E)

Intramuscular triamcinolone Oral amoxicillin Oral doxycycline Topical hydrocortisone Topical ketoconazole

1

USMLE Step 2 Assessment Exam

3.

An 8-year old boy is brought to the emergency department because of wheezing. According to the parents, the child has “always” had asthma and uses an albuterol inhaler at home when he has wheezing. This episode started 2 days ago but did not respond to several doses of inhaled albuterol. He has been coughing frequently and vomited twice in the previous day. Examination shows that he has diffuse wheezing with decrease in breath sounds on both sides, and prefers to sit up in a tripod-like position. Vital signs are: pulse 142/min, respirations 38/min, blood pressure 108/72 mm Hg, and temperature 37.2 C (99.0 F). Pulse oximetry shows that the oxygen saturation is 89%. Which of the following is the most appropriate first step in management? (A) (B) (C) (D) (E)

4.

2

Albuterol by nebulizer Chest x-ray Intravenous fluids Methylprednisolone Oxygen by mask

A 16-year-old girl is brought to the physician because of “swollen glands” on the neck that were first noticed 3 days earlier. The girl is otherwise feeling well and denies sore throat, fever, chills, and headaches. Her past medical history is unremarkable, and she does not take any medications on a regular basis. The family history is also noncontributory and no other family members have similar disease. On physical examination the patient does not seem to be in any acute distress. Her vital signs are within normal limits. On the right side of her chin three faintly pink 3-mm papules are noted. When specifically questioned about the papules she reports that these “pimples” arose a couple of weeks earlier after her pet kitten scratched her, but that they are now almost resolved. Inspection of the neck is significant for marked swelling of the right side below the jaw line, and palpation reveals enlarged, tender, mobile lymph nodes in that area. Which of the following studies is indicated at this time? (A) (B) (C) (D) (E)

5.

Nasopharyngeal swab for culture Needle biopsy of the lymph nodes No further studies are indicated Radiography of the head and neck Serum antibody titers

A previously healthy 6-year-old girl has a low-grade fever, malaise, and a new-onset rash on her face and arms of 2 days’ duration. She attends kindergarten but the parents are not aware that any other children have been reported sick. They have a 5-month-old son also, but he has not shown any signs of illness. The family history is unremarkable. On physical examination the patient seems to be in mild distress. Her temperature is 37.3 C (99.1 F), pulse is 120/min, and respirations are 28/min. Physical examination shows hyperemic oropharyngeal mucosa and edematous tonsils with scant clear secretion. The patient’s face has bright red erythema of the cheeks bilaterally. There is a reticulated, erythematous exanthem on the trunk and upper extremities. The palms and soles are unremarkable. Which of the following is the most likely etiologic agent of this patient’s disease? (A) (B) (C) (D) (E)

6.

Coxsackievirus A16 Herpes simplex virus type 1 Human herpesvirus type 6 Parvovirus B19 Rubella virus

A 7-month-old infant is brought to the clinic for a follow-up visit. Two days ago the infant was taken to the emergency department because of a large bruise over his thigh without any obvious history of trauma. Coagulation tests had revealed that the factor VIII level was less than 1% (normal 70 to 130%), which suggested a diagnosis of hemophilia A. In the intervening period, there had been no further episodes of bruising or bleeding. On today’s visit the infant appears active and playful, with no pain in the affected extremity. History reveals that there are two other siblings in the family, both female, who have no bleeding tendency. Neither of the parents has history of excessive bleeding, but one of the maternal uncles has hemophilia A. The mother is pregnant with her fourth child now, but is worried that the baby may be born with hemophilia A. In advising the parents, which of the following is the best estimate of the risk of hemophilia in the next child? (A) 25% for a female offspring, 50% for a male offspring (B) 25%, regardless of the offspring’s sex (C) None if it is a female offspring, 50% for a male offspring (D) No risk if the father is normal (E) Risk cannot be determined unless mother’s carrier status is determined

Pediatrics

7.

A 4-year-old boy is brought to the emergency department because of fever, malaise, and photophobia of 4 days’ duration. The child’s illness began with a runny nose and loss of appetite but has worsened progressively despite the use of over-the-counter cold medication. This morning his parents noticed a rash on the head and neck. The child was adopted recently from an orphanage in Eastern Europe and was in good health when he arrived in the United States 2 weeks earlier. The parents had been assured there was no significant past medical or family history. Apart from seeming slightly malnourished, the couple had not noticed any health problems. They are not sure if he had received all the necessary immunizations. His temperature is 40.1 C (104.2 F). He is quite bothered by the examination light and has significant conjunctival injection bilaterally. There are bluish-gray specks located on the buccal mucosa. There is a maculopapular erythematous rash on the forehead and behind the ears, descending on the sides of the neck. Administration of which of the following vaccines most likely would have prevented this disease? (A) (B) (C) (D) (E)

8.

Hepatitis B vaccine Influenza vaccine Measles-mumps-rubella vaccine Meningococcal vaccine Varicella vaccine

A 7-year-old girl is brought to the clinic for a routine school physical examination. The child has been regularly followed in the same clinic, and all previous visits were uneventful. She has demonstrated normal gain in weight and height, and her school performance in the previous year was satisfactory. At today’s visit, both the height and weight are at the 75th percentile. Her blood pressure is normal and there is no cardiac murmur. Lungs are clear and the abdomen shows no palpable masses. Genitourinary examination reveals normal female genitalia, with no clitoromegaly. There is sparse growth of pubic hair. There is no facial or axillary hair, and breast development is absent. The mother is concerned about the appearance of pubic hair in her daughter. Which of the following best describes this patient’s condition? (A) (B) (C) (D) (E)

Congenital adrenal hyperplasia Exogenous androgen effect Normal puberty Premature adrenarche Turner syndrome

9.

A previously healthy 12-month-old girl is brought to the clinic because of sudden-onset high fever and runny nose. She became very fussy and irritable the prior evening and spent most of the night crying. She has not eaten well since the illness began. The patient goes to daycare where several other children have been reported ill in the prior week. Her temperature is 40.3 C (104.5 F), pulse is 130/min, and respirations are 30/min. Physical examination reveals bilateral conjunctivitis, rhinorrhea with edematous nasal mucosa and clear secretions, and palpable occipital lymph nodes. Symptomatic treatment is recommended. Two days later the fever and runny nose have suddenly ceased but the child has broken out in a generalized skin rash consisting of hundreds of pinkish-red spots over almost the entire skin surface. Which of the following is the most likely diagnosis? (A) (B) (C) (D) (E)

10.

Measles Roseola infantum Rubella Scarlet fever Varicella

A 14-year-old girl with asymptomatic skin lesions is referred to the dermatologist from a psychiatric hospital where she was being evaluated for mental retardation and emotional problems. There is no record of a seizure disorder. The family history is unavailable because the child was adopted. She is on no medication and denies allergies to medication. On physical examination, the patient is in no acute distress. Her blood pressure is 100/50 mm Hg, pulse is 85/min, and respiratory rate is 16/min. Neurologic examination reveals a cooperative girl with mild mental retardation and low affect. Inspection of the skin is significant for multiple homogenously brown, well-demarcated patches 2 to 5 cm in diameter and several pedunculated, soft, flesh-colored papules on the right inner thigh. Funduscopic examination is normal. Mild scoliosis is detected on skeletal exam. A complete blood count with differential white count was performed on admission to the psychiatric institution and was within normal limits. Which of the following radiologic findings is most likely to be present? (A) (B) (C) (D) (E)

Jaw cysts Osteopoikilosis Osteopathia striata Thinning of the long-bone cortex “Train track” calcifications of the skull

3

USMLE Step 2 Assessment Exam

11.

A 5-year-old boy has dysuria, fever, leukocytosis, and right flank pain. On physical examination he is tender to fist percussion over the right costovertebral angle. Urinalysis shows grossly infected urine. He is treated with appropriate antibiotics and the infection clears. This is verified by repeated negative urinary cultures. The family indicates that he has had two previous similar episodes that were also successfully treated with antibiotics. No further workup was done following those two episodes. The reason for this recurrent problem is most likely to be diagnosed with which of the following studies? (A) (B) (C) (D) (E)

12.

Cystoscopy Intravenous pyelogram Renal ultrasound Retrograde urethrogram Voiding cystourethrogram

A previously healthy 12-year-old Middle Eastern boy is brought to the office because of low-grade fever and a skin rash of 2 days’ duration. He complained of a sore throat and mild neck pain several days earlier. The entire family immigrated to the United States a year earlier as refugees from a war-torn country. This is the child’s first visit to a physician since then. No other family members have been affected with the illness; however, they live in close quarters with several other relatives and are concerned that the infection might spread. The boy’s temperature is 39.2 C (102.6 F), pulse is 90/min, and respirations are 22/min. He is in the fiftieth percentile for height and weight. Inspection of the nasopharynx reveals mild hyperemia and a serous exudate on the posterior pharyngeal wall. Faint erythematous macules are present on the soft palate. He has tender, enlarged posterior cervical and posterior occipital lymph nodes. In addition, there are many erythematous macules and papules on the face, chest, and proximal extremities. Who of the following would be most likely to develop complications from contact with this patient while he is infectious? (A) (B) (C) (D)

His 4-month-old niece His 15-year-old sibling His 23-year-old pregnant aunt His 60-year-old grandmother with systemic lupus erythematosus (E) His 75-year-old grandfather with diabetes mellitus

4

13.

A 13-year-old boy has been complaining of pain just below the knees for the past several months. He plays for the school basketball team and practices regularly. There is no history of fever or change in weight, appetite, or level of energy. There is no family history of rheumatic illness. Examination reveals that he has normal weight and height and is in Tanner stage III of pubertal development. Knee examination shows that the tibial tuberosities are prominent and tender on both sides. The movements of the knee are unrestricted on both sides and there is no redness or swelling of the joint. Radiographs reveal mild elevation of the tibial tuberosity, but are otherwise normal. Which of the following is appropriate management? (A) (B) (C) (D) (E)

14.

Obtain a complete blood count Obtain an erythrocyte sedimentation rate Obtain a magnetic resonance imaging of the knees Reassure the family that the condition is self-limiting Refer the patient for orthopedic consultation

A 2-week-old boy is brought to the clinic for routine checkup. The infant was born at full term after a normal pregnancy by normal vaginal delivery, and the birth weight was 7 lb (3.15 kg). He was breast-fed from birth and the mother reports that he was feeding well for the first 10 days, but lately he has become fussy and often interrupts feeding. The infant’s weight today is 7 lb, 8 oz (3.38 kg) and he seems comfortable. His temperature is 37.2 C (98.9 F), and pulse and respiratory rate are normal. Examination of the mouth reveals white plaques on the buccal mucosa, tongue, and palate, which on removal leave behind punctate bleeding spots. The lung and heart examinations are normal. There is no evidence of hepatosplenomegaly. There is no rash on the perineum or genital areas. Which of the following is the most appropriate management? (A) (B) (C) (D) (E)

Amoxicillin therapy Amphotericin B therapy Change to formula feeding No treatment is required Nystatin therapy

Pediatrics

15.

A 2-year-old boy is brought to the emergency department because of noisy breathing for the last 2 days. He was well until 3 days ago when he developed a fever up to 38.5 C (101.3 F), cough, and runny nose. One day later, the breathing became noisy with a loud sound during inspiration, which is more prominent during night. He is able to eat and drink fluids and has no vomiting. There is no previous history of similar episodes or chronic cough or choking. On examination, the child has mild respiratory distress with a loud inspiratory stridor. He is alert and afebrile, and oxygen saturation is 98% in room air. Examination reveals good breath sounds on both sides and no wheezing. There is no cardiac murmur and abdominal examination is normal. Chest x-ray shows that the subglottic space is narrowed and resembles a pencil point. Which of the following is the most likely causal organism? (A) (B) (C) (D) (E)

Haemophilus influenzae Parainfluenza virus Respiratory syncytial virus Streptococcus group A Streptococcus pneumoniae

16.

A 2-year-old boy is brought to the physician for a routine health maintenance examination. He was previously followed at another clinic, but due to change in medical insurance he had to switch physicians. He was born at full term with a birth weight of 7 lb, 12 oz. He was fed an iron-containing formula from birth and since the age of 1 year he is on whole cow’s milk. He also eats various other foods that the mother cooks for the family. He is able to join two words together and has a vocabulary of well over 50 words. He was never sick enough to require hospitalization. At the 1-year visit, he was found to have hemoglobin of 10.2 g/dL. He was put on oral iron (ferrous sulfate drops), which the mother administered regularly. However, 2 months later his hemoglobin was still 10.3 g/dL. The dose of iron was increased and the hemoglobin was rechecked 3 months later and found to be unchanged. Thereafter, the mother had continued the ferrous sulfate drops until this visit. Today a complete blood count shows: Hemoglobin

10.3 g/dL

Hematocrit

31%

Mean corpuscular volume

62 fL

RDW

13.1

Reticulocytes

1.8%

Leukocyte count

10.5 × 109/L

Differential:

Neutrophils 35%, lymphocytes 54%, monocytes 8%, eosinophils 1%

Platelet count

280 × 109/L

Which of the following studies is most likely to reveal the cause of his anemia? (A) (B) (C) (D) (E)

Bone-marrow aspiration Hemoglobin electrophoresis Serum B12 level Serum iron and ferritin level Upper and lower gastrointestinal endoscopy

5

USMLE Step 2 Assessment Exam

17.

A 16-year-old boy has a milky white discharge from his urethra, most prominent in the morning when he wakes up. There is also a mild burning sensation when he urinates. It started approximately 2 weeks earlier and is not showing any signs of improvement. He is in good general health and has no other concerns. He is sexually active but does not have a steady sexual partner. He does not use a condom regularly. In the previous 6 months he had sexual relations with four different women. On physical examination, the patient is a welldeveloped young man in no acute distress. His vital signs are within normal limits. Inspection of the genitals reveals mild erythema of the urethral orifice and a purulent discharge that is easily expressed with manual pressure. The inguinal lymph nodes are not palpable. There do not appear to be any lesions present on the penis or adjacent skin. A urethral swab is most likely to show which of the following findings? (A) Colorless pyriform flagellates in a saline smear (B) Chains of Gram-negative bacilli in macrophages (C) Gram-negative intracellular diplococci in polymorphonuclear cells (D) Motile spiral spirochetes on darkfield microscopy (E) Multinucleated giant cells in a Tzanck smear

18.

A full-term newborn infant girl is noted to be cyanotic and in severe respiratory distress. She has no breath sounds on the left and cardiac sounds are heard only faintly on the left side. Breath sounds can be heard on the right, but they are obscured by prominent heart sounds on that side. The abdomen is scaphoid. X-ray shows the presence of bowel loops on the left hemithorax and deviation of the mediastinal structures to the right. Which of the following is the most appropriate next step in management? (A) Emergency surgical intervention (B) Endotracheal intubation and mechanical ventilation with supplemental oxygen (C) Insertion of a chest tube on the left (D) Pharmacologic agents to increase the pulmonary vascular resistance (E) Positive pressure ventilation through a face mask

6

19.

A 16-year-old boy comes to the physician because of a nonhealing ulcer on the left heel that has been present for 3 months. The family has recently moved to the area from the West Coast and his medical records have not yet been transferred. He has had problems with his legs since he was about 10 years old—it started as vague weakness and increased clumsiness but progressed over the years to the point that he now needs a cane to walk and special orthotics to support his normal weight. The family history is significant for similar disease in multiple members of his mother’s family. He is currently taking only nonsteroidal antiinflammatory drugs for muscular pain, and he denies allergies to any medication. On physical examination, the patient is in no acute distress. His vital signs are within normal limits and a basic chemistry panel and complete blood count performed a month earlier show no abnormalities. Inspection of the patient reveals considerable leg muscle wasting with a stork-leg appearance. There is pronounced bilateral pes cavus deformity and thoracic scoliosis. Deep tendon reflexes are absent on the lower extremities bilaterally and proprioception is significantly decreased. You note the presence of essential tremor while examining the patient. On the hands there are palpable enlarged peripheral nerves. A small, shallow, clean ulcer on the right heel corresponds to a rough protruding part on his right prosthesis and you assure him that getting a new prosthesis will expedite the healing process. Which of the following is the most likely diagnosis? (A) (B) (C) (D) (E)

Charcot-Marie-Tooth disease Duchenne muscular dystrophy Friedrich ataxia Guillain-Barré syndrome Werdnig-Hoffman disease

Pediatrics

20.

21.

A 20-year-old pre-med student comes to the clinic because of a painful outbreak on his genitals that started 2 days earlier. He also complains of malaise and lowgrade fever that is partially responsive to aspirin. Reluctantly, he admits that about a week before this illness he had sexual intercourse with a woman he met at a local fraternity party. He used a condom but noticed after their encounter that it had ripped at the tip. He is in good overall health and works out on a regular basis. On physical examination, he is a well-developed, fit young man with a temperature of 37.9 C (100.2 F) and a pulse of 90/min. His respiratory rate and blood pressure are within normal limits. Inspection of the genitals reveals multiple tense, grouped vesicles with an erythematous base on the glans and shaft of the penis. There are also several painful small ulcerations. The inguinal lymph nodes are symmetrically enlarged bilaterally, and tender to palpation. Which of the following diagnostic studies is indicated at this time?

22.

(A) (B) (C) (D) (E)

23.

Bacterial culture Complete blood count KOH preparation Tzanck preparation Skin biopsy

A 1-year-old child is brought to the pediatric emergency department by paramedics. The child was in the care of his baby-sitter, who reports she found the infant lethargic in his playpen. She denies that the child could have swallowed any medication or other substance. The child’s parents are currently not available to provide any information. The infant is semi-arousable and irritable when examined. There is soft tissue swelling over the right parietal area, subconjunctival hemorrhages bilaterally, and multiple ecchymoses over the back and upper chest. Which of the following is the most appropriate next step in management?

A newborn infant who was born full-term is discharged home from the hospital with his mother on day 3 of life. At age 1 week the infant is seen by a pediatrician for his first newborn visit. Nutritional intake has been adequate and the infant has been gaining weight appropriately. Physical examination is normal. As part of the routine behavior assessment, the physician educates the infant’s mother about what she can expect from the child developmentally. Which of the following will most likely be found in this infant? (A) (B) (C) (D)

Absence of grasp reflex Absence of rooting reflex Asymmetric tonic neck reflex Long latency between a painful stimulus and a cry

(E) Rolls over from back to stomach (F) Turns head toward human voice

A 16-year-old girl is brought to the emergency department by paramedics on a Saturday night. She is not accompanied by a family member, and is lacking available history. Physical examination reveals delirium, dilated pupils, dry and flushed skin and mucous membranes, peculiar myoclonus-like twitching, tachycardia, and fever. Her clinical condition appears to be deteriorating during a period of 15 minutes of observation. The attending physician suspects that the patient is having a reaction to an ingested drug. Which of the following medications would be most appropriate to treat this patient? (A) (B) (C) (D) (E)

Edrophonium Phenobarbital Phenothiazine Physostigmine Thiamine

(A) Await parental consent before obtaining any necessary diagnostic procedures on the child (B) Confront the baby-sitter (C) Consult a social worker first thing in the morning (D) Protect the child’s confidentiality at all costs (E) Report the suspicion to the appropriate state agency immediately

7

USMLE Step 2 Assessment Exam

24.

A 2-year-old boy has a cough and occasional wheezing for the past 3 weeks. Initially, his mother gave him an over-the-counter cough mixture but there was no improvement. He has had no fever, runny nose, or choking episodes during this time. He has been active with good appetite and does not seem to have lost any weight. He has a normal birth history and no previous episodes of coughing. There is no family history of asthma, tuberculosis, or cystic fibrosis. There is one older sibling, a 5-year-old sister who is healthy. Examination reveals an active, playful child in no distress. Throat, nose, and ears look normal. Chest examination reveals diminished breath sounds on the right side at the apex, and an intermittent wheeze is heard on the right side. Cardiac examination is normal and there is no enlargement of the liver or spleen. Extremities are normal with no clubbing of the fingers. Which of the following investigations is most likely to reveal the etiology of his cough? (A) (B) (C) (D) (E)

8

Barium swallow Chest x-ray Esophageal pH probe study Skin allergen testing with house dust mite antigen Sweat chloride test

25.

A 4-month-old girl is brought to the physician for a follow-up visit. She was delivered by cesarean section at 40 weeks’ gestation after a high-risk pregnancy that was marked by reduced movements in utero and reduced spontaneous activity after birth. The Apgar scores were 5 and 6 at 1 and 5 minutes, respectively. After birth, the patient showed signs of bulbar weakness, with sucking and swallowing difficulty and a weak cry. In resting position, she assumed a jug-handle position of the arms and frog-leg position of the legs due to proximal symmetrical muscle weakness. At age 4 months, the physical examination shows a poorly active infant in the 8th percentile for height and weight. The arms are flaccid, with abducted and internally rotated shoulders. Her hips are abducted, with flexed knees and externally rotated feet. She cannot hold her head when in upright position because of spinal muscle atrophy. Inspection of the mouth reveals fasciculations of the tongue. The deep tendon reflexes are absent. Her parents are very concerned about her condition. They should be told which of the following? (A) (B) (C) (D)

It is associated with severe mental retardation It is inherited in an autosomal recessive trait The gene mutation is located on the X-chromosome Serum creatine kinase levels are the diagnostic test of choice (E) She will never attain normal sphincter function

Pediatrics

26.

An infant is born to a 27-year-old primigravid mother after a full-term pregnancy. The delivery and resuscitation were uneventful and the Apgar scores were 7 and 10 at 1 and 5 minutes, respectively. During the newborn physical examination, the baby is discovered to have a heart murmur. He has been feeding well since birth and there is no respiratory distress or enlargement of the liver. Other features noted at this time are small ears and a prominent tongue. Echocardiogram reveals the presence of atrioventricular canal defect. There is no family history of congenital malformations or mental retardation. In planning further tests to establish a diagnosis, it is most appropriate to mention to the parents that the infant might have (A) (B) (C) (D) (E)

27.

DiGeorge syndrome Prader-Willi syndrome Trisomy 13 Trisomy 18 Trisomy 21

A 2-month-old male infant is brought to the emergency department with wheezing and rapid breathing for the last 2 days. He was born at full term with a birth weight of 7 lb, 2 oz, and was well until 2 days ago when he developed a fever of 38.8 C (101.8 F) and runny nose. He began to cough and then became “wheezy.” He has been irritable and unable to feed from the bottle without pausing several times, and has vomited twice. There is a 4-year-old elder brother who has had a runny nose for the past week. Vital signs are: pulse 146/min, respirations 56/min, blood pressure 84/58 mm Hg, temperature 37.3 C (99.1 F), and oxygen saturation (by pulse oximetry) 96% on room air. Examination shows an active and alert baby in mild respiratory distress. The nose is blocked with clear secretions, while the throat and ear examination is normal. He has slightly reduced breath sounds on both sides and bilateral rhonchi. Cardiac examination is normal and there is no hepatosplenomegaly. Which of the following studies is most likely to reveal the etiology of the illness? (A) (B) (C) (D) (E)

28.

Bronchoscopy Chest x-ray Complete blood count Nasopharyngeal aspirate Throat swab

A 3-year-old boy is brought to the emergency department because of drooling. He developed a sore throat 3 days ago with fever up to 38.8 C (101.8 F). His mother gave him acetaminophen and an over-the-counter mixture for what she assumed was a common cold; however, since last night the fever had increased to 40 C (104.0 F) and the child has become increasingly unwell. He did not want to drink and his respirations were becoming noisy. When he woke up this morning he began drooling and appears to be in pain. There was no history of trauma, cough, or vomiting, and his immunizations are complete to date. Examination reveals a toxic and frightened-looking child who refuses to flex his neck. Vital signs are: temperature 40.3 C (104.5 F), pulse 148/min, respirations 32/min, blood pressure 110/68 mm Hg, and oxygen saturation 98% in room air. There is a bulge in the posterior pharyngeal wall, along with erythema and collection of oral secretions. Which of the following investigations would be most helpful in establishing a diagnosis? (A) (B) (C) (D) (E)

29.

Complete blood count Lateral neck x-ray Lumbar puncture Nasopharyngeal aspirate Throat swab

A 1-day-old newborn has developed an erythematous rash. The infant was born by normal vaginal delivery to a 26-year-old primigravid mother at full term with a birth weight of 8 lb, 3 oz. The delivery was uneventful and the Apgar scores were 8 and 10 at 1 and 5 minutes, respectively. The pregnancy was supervised and uncomplicated, with negative serology for syphilis and toxoplasmosis. Since birth the baby was formula fed and he has passed urine and meconium. Examination shows that the infant is alert looking, with good tone and normal Moro reflex. There is a blotchy erythematous rash on the face, chest, and back. A closer look at the rash reveals small vesicles at the center of the erythematous areas. The chest and lung examination is normal and there is no hepatosplenomegaly. Microscopic examination of the vesicular fluid reveals predominance of eosinophils. Which of the following is most likely to be the cause of this infant’s rash? (A) (B) (C) (D) (E)

Allergy to cow’s milk Atopic dermatitis Erythema toxicum neonatorum Intrauterine infection Sepsis and disseminated intravascular coagulation

9

USMLE Step 2 Assessment Exam

30.

A 3-week-old infant is brought to the clinic because of a fever. He was born at full term after a normal pregnancy and weighed 7 lb, 12 oz at birth. The delivery was uneventful and he was discharged from the hospital after 48 hours. He has been formula fed and was doing well until the day before, when the fever was first noticed. Several family members have had a viral upper respiratory infection in the past week, including the mother. Upon examination, the temperature is 38.9 C (102.0 F). The baby appears awake and has good tone in the extremities. Respiratory, cardiovascular, and abdominal examinations are within normal limits. The eardrums are normal and there is no skin rash. In the office he is offered a bottle and is able to feed 3 oz of formula. Which of the following is the most appropriate management?

32.

(A) Acetaminophen infant drops for fever control and monitoring at home (B) Arrange for another office visit in 2 days’ time (C) Refer to emergency room for urgent admission (D) Send for a complete blood count and, if normal, reassure mother that baby has a viral infection (E) Start oral amoxicillin

31.

A 14-year-old girl is brought to the clinic for a routine school physical examination. The mother is concerned that her daughter has shown no signs of breast development and has not started menstruating. Past history does not reveal any significant illnesses. Review of the patient’s growth reveals she was always just below the 5th percentile until she was 10 years old. However, her current height is well below the 5th percentile with an absence of pubertal growth spurt. Physical examination reveals a short girl with normal facial appearance. There is a prominent cubitus valgus deformity. The neck is short and the posterior hairline is low. There is no breast development, although some pubic and axillary hair is present. Examination of the heart, lungs, and abdomen reveals no abnormality. Which of the following studies is most likely to reveal the etiology of this patient’s short stature? (A) (B) (C) (D) (E)

10

Abdominal ultrasound Gonadotropin level Growth hormone level Karyotype Thyroid hormone level

A newborn infant is diagnosed with dehydration and is transferred to the neonatal intensive care unit. The infant was born at 38 weeks’ gestation to a healthy 26-year-old mother. Prenatal care was good, there were no complications during pregnancy, and the delivery was by way of normal, spontaneous, vaginal delivery. Initially the infant seemed well; however, soon after there was poor oral feeding, vomiting, and lethargy. On physical examination in the NICU, the infant is lethargic and has a weak cry. Vital signs are systolic blood pressure 40 mm Hg, pulse 190/min, and respirations 25/min. The abdomen is soft and there are no masses palpable. Examination of the genitalia reveals an enlarged phallus with significant curvature of the phallus. The urethral meatus is in a hypospadiac location. The scrotum is rugated, and the testicles are nonpalpable bilaterally. At this time, which of the following is the most important information to obtain to treat the patient? (A) (B) (C) (D) (E)

33.

Arterial blood gas Karyotype Pelvic ultrasound Serum electrolytes Serum 17-alpha-hydroxyprogesterone level

A 21-year-old primigravid woman comes to the physician at 12 weeks’ gestation. Her brother and sister are both affected with Disease X. Her mother also had the disease but her father was not affected. Her affected brother has three children, two of which are girls and both of which are affected with the disease. Her nephew does not have Disease X. Which of the following diseases is consistent with the described inheritance pattern? (A) (B) (C) (D) (E)

Albinism Congenital adrenal hyperplasia Bruton agammaglobulinemia Hypophosphatemic rickets Duchenne muscular dystrophy

Pediatrics

34.

A 23-year-old primigravid woman with a history of alcohol and substance abuse is admitted to the hospital in true labor, showing signs of imminent delivery. She is transferred to the delivery room, and 30 minutes later a full-term baby girl is born. The new mother has not had any prenatal care because she “never had any problems during the pregnancy.” Examination of the mother shows signs of severe malnutrition. The newborn is 42 cm long and weighs 2,500 g. She scored 6 and 7 on the Apgar scale at 1 and 5 minutes, respectively. Physical examination of the neonate does not reveal any signs of acute distress. Her vital signs are within normal limits. However, physical examination reveals flaccid paralysis of the lower extremities, subluxed hips, and bilateral clubfoot deformity, and examination of the back reveals a large cystic sac protruding from the midline lumbosacral region. Deep tendon reflexes are absent on the lower extremities. Which of the following nutritional supplements could have prevented this defect? (A) Folic acid (B) Vitamin A (C) Vitamin B2 (D) Vitamin B6 (E) Vitamin K

35.

A 9-year-old boy is brought to the physician for evaluation of what the parents complain of as “increasing clumsiness.” He used to be very athletic, participating in multiple sports, but has recently dropped out because he cannot keep up with the other children. The parents had also noticed a change in his speech, with difficulty expressing himself to the point that on several occasions he almost choked while talking. His prior medical history is unremarkable and he is on no medications. There is no family history of similar disorders. On physical examination, the child is in no distress and has normal vital signs. He is in the 50th percentile for height and weight. Neurologic evaluation reveals a wide-based gait with constant shifting of position to maintain balance. Sitting and standing are associated with titubation. When he walks there is uneven and irregular striking of the floor by his soles, with wild movements produced in trying to correct the imbalance. His speech is a little slurred and slow. Extensive blood work did not reveal any significant abnormality. An echocardiogram showed symmetric, concentric, ventricular hypertrophy. Which of the following is the most likely diagnosis? (A) (B) (C) (D) (E)

Friedrich ataxia Refsum disease Spastic hemiplegia Werdnig-Hoffman disease Wilson disease

11

USMLE Step 2 Assessment Exam

36.

A 9-month-old boy is brought to the physician because the mother noticed “orange sand” in his diapers over the previous several days. The child has been in good health, although a little slow to develop his milestones. He has no significant past medical history and takes no medication. The family history is unremarkable. On physical examination, the patient is in no acute distress. His temperature is 36.7 C (98 F), pulse is 95/min, and respirations are 20/min. The weight and length are in the 20th and 10th percentiles, respectively. He sits with assistance and lifts his upper body when in a prone position, but does not attempt to crawl or stand. Involuntary movements indicative of extrapyramidal dysfunction are noted. There are prominent calluses on both hands and you note that the child repeatedly gnaws on those areas. A complete blood count and chemistry panel reveals macrocytic anemia and hyperuricemia. In addition, which of the following studies is most likely to be abnormal in patients with this condition? (A) (B) (C) (D) (E)

Cerebrospinal fluid analysis Computerized tomography of the brain Magnetic resonance imaging of the brain Plain abdominal radiograph Ultrasound of the abdomen

37.

A 16-year-old girl is brought to the emergency department by her parents because of nausea, vomiting, and malaise that have been progressively worsening over the course of several days. They also noted some speech changes and a fine tremor most prominent when she tried to complete fine tasks, such as threading a needle. They first thought she was on one of her crash diets to lose weight because she would not eat anything, but when she finally complained of feeling sick they decided to have her examined. She has no significant past medical history and her menstrual periods are regular. She denies allergies to medication and intake of any prescription or over-the-counter drugs in the previous month. The family history is significant for a paternal uncle who died at a young age of liver cirrhosis. On physical examination, the patient is in moderate distress. Her pulse is 110/min, respirations are 24/min, and blood pressure is 90/45 mm Hg. Inspection of the skin reveals multiple telangiectatic arterioles in the skin with radiating capillary branches, redness of the palms, and pronounced mild jaundice. There is a hint of digital clubbing. A tangreen color is seen in the limbic area of the eye. Laboratory studies reveal a ceruloplasmin level of 17 mg/dL and low total serum copper. Which of the following studies is necessary to confirm the diagnosis? (A) (B) (C) (D) (E)

12

Analysis of 24-hour urine collection Computerized tomography of the brain Liver biopsy Skeletal radiologic survey Slit-lamp examination of the eye

Pediatrics

38.

A 12-year-old white girl has right distal thigh pain for the past several months that has worsened over the past few days. She recalls that she was struck in gym class by a ball in the affected thigh and that caused a period of excruciating pain. She also feels that her right thigh is slightly larger than her left and that her right calf and foot sometimes swell. She is a well-developed and well-nourished girl. The patient’s vital signs are all within normal limits. On examination of the distal right thigh, there is a noticeable difference in circumference of the right thigh versus the left. Range of motion is decreased with respect to the knee secondary to pain. There is 1+ edema to the mid-calf level and distal pulses are normal and symmetric. Radiographs of the distal femur show a bone-forming mass with rays of bone formation radiating from the center of the mass. The periosteum is raised off the bone by the mass, causing fusiform swelling with reactive periosteal bone at the periosteal margin. Which of the following is the most likely diagnosis? (A) (B) (C) (D) (E)

Benign bone cyst Fracture of the distal femur with callous formation Osteomyelitis of the distal femur Osteosarcoma Paget disease

39.

A 20-month-old girl is brought for evaluation. The parents complain that she has been behaving in a peculiar way over the past couple months, as though she was forgetting the skills she had already mastered earlier. Her vocabulary seems to have shrunk, and she has become very placid compared to other children she spends time with. The child has lost interest in playing with her friends and is often found sitting alone, wringing or clapping her hands repeatedly. She no longer makes eye contact with her parents. The most recent development is the occurrence of breath-holding or rapid breathing while she is awake. She sleeps peacefully, though. Her prior medical history, as well as the family history, is unremarkable. On physical examination, the patient is in no acute distress and her vital signs are within normal limits. Her height, weight, and head circumference are in the 20th, 30th, and 5th percentiles, respectively. During the examination, she sits quietly on the examining table and does not make eye contact. Repeated hand wringing and general hypotonia are noted. A Wood lamp examination of the skin is negative. Basic laboratory studies and neuroimaging studies reveal no abnormality. Genetic testing is offered to the family. Which of the following genes is most likely to be sequenced in this patient? (A) (B) (C) (D) (E)

MECP2 NF1 PTEN SPINK5 TSC1

13

USMLE Step 2 Assessment Exam

40.

The parents of a 2-week-old child bring the newborn to the clinic for a routine visit. At birth the infant was noticed to have a cleft lip. Both parents are 24 years old and this is their first child. The pregnancy and delivery were uneventful and, apart from the lip, no other congenital malformations were noted. The birth weight was 7 lb, 4 oz, and at this visit the infant has gained 4 oz on formula feeds. Examination reveals an active newborn with unremarkable examination findings. The mother mentions that she has heard about recurrence of congenital abnormalities in subsequent offspring. The parents want to know the risk of having a second child with cleft lip. At this time, which of the following is the best advice?

42.

(A) Chromosomal testing from the infant and both parents is required to estimate the risk (B) (C) (D) (E)

41.

The recurrence risk of cleft palate is 5% The risk of recurrence is 25% The risk to the second child is not increased The risk will increase with increasing maternal age

A 12-year old boy is brought to the emergency department due to worsening of asthma. He has a history of several previous admissions due to status asthmaticus. At home he uses inhaled steroids on a regular basis. He started to wheeze 3 days ago and used an albuterol inhaler, which provided partial relief. In the last several hours before coming to the hospital he developed increasing difficulty in breathing and could not talk in full sentences. His vital signs are: pulse 148/min, respirations 32/min, blood pressure 126/74 mm Hg, and temperature 37.8 C (100.0 F). He appears confused and disoriented, lung examination reveals markedly diminished breath sounds, and pulse oximetry shows an oxygen saturation of 87%. He is started on oxygen and nebulized albuterol. Arterial blood gas analysis shows: pH

7.29

pO2

84 mm Hg

pCO2

44 mm Hg

HCO3

24 mEq/L

Which of the following is the most appropriate next step in management? (A) (B) (C) (D) (E)

14

Administer intravenous fluids Administer methylprednisolone Administer sodium bicarbonate Admit for observation Intubate and provide mechanical ventilation

A 5-year-old boy is brought to the physician for evaluation. The parents complain that he has trouble maintaining his balance and has started walking in a peculiar, waddling way as of lately. He pushes his knees when he wants to stand up and has developed large calf muscles that seem to stand out from the rest of his body. His medical history is significant for several episodes of pneumonia. He is on no medications at this time and has no allergies to medication. The family history is unremarkable. On physical examination, the child is in no acute distress. His vital signs are within normal limits. He has difficulty generating a forceful cough for chest auscultation. Inspection of the trunk and extremities reveals very pronounced calf muscles. Deep tendon reflexes are significantly diminished. There is mild kyphoscoliosis secondary to paraspinal muscle weakness. Which of the following studies is most likely to aid in confirming the diagnosis? (A) (B) (C) (D) (E)

43.

Electrocardiogram Electromyography Muscle biopsy Radiographs of the spine Serum creatinine phosphokinase

A 9-month-old male infant is brought to the clinic because of wheezing and a cough. He has been taken to the emergency room three times in the past 5 months for “breathing problems.” On each visit he responded to inhaled bronchodilator therapy. Recently, though, the wheezing and coughing episodes are becoming more frequent. Also, the infant has developed loose stools, which are greasy and foul smelling. His weight today is 16 lb, and the infant’s growth chart shows loss of growth percentiles in the last 3 months. Examination reveals a smiling infant in mild respiratory distress. The pulse is 128/min, respirations are 42/min, and oxygen saturation is 98% on room air. Chest examination reveals an increased anteroposterior diameter and bilateral wheezing. There is no cardiac murmur and the abdominal examination shows no hepatosplenomegaly. Which of the following studies is most likely to reveal the etiology of the infant’s illness? (A) (B) (C) (D) (E)

Bronchoscopy Chest x-ray Lung biopsy Sweat chloride testing Upper and lower gastrointestinal endoscopy

Pediatrics

44.

A physician volunteers to participate in a screening program at the local school organized to detect head lice in their students. The program is conducted at the school, where the physician systematically examines the head and scalp hair of all third-graders present. During the examinations, there is a 9-year-old girl with thick, long, curly blonde hair that has abundant white, gritty scale on the retroauricular and posterior occipital scalp with several enlarged, nontender suboccipital lymph nodes. There is also patchy red lichenification and excoriations on the posterior neck. Despite the best effort to characterize the nature of the white particles, which appear to be attached to hair shafts, the physician is unable to make a conclusive decision. Which of the following may prove helpful in making the correct diagnosis in this patient? (A) (B) (C) (D) (E)

45.

Culture on Sabouraud’s agar Hair penetration test Microscopic examination of the snipped hair Potassium hydroxide hair preparation Pull test

A concerned mother brings her 2-month-old infant to the clinic for a routine appointment. The child is healthy and the mother has no medical complaints at this time. He has begun to play with a rattle, can hold his head erect temporarily, and often coos and smiles when played with. One of the mother’s friends lost a similarly healthy 2-month-old infant to sudden infant death syndrome (SIDS). Understandably the mother wishes to do everything possible to protect her child while he is sleeping. She has heard conflicting information regarding sleeping habits and positions that can possibly reduce the risk for SIDS. She should be advised that her infant should sleep in which of the following positions? (A) (B) (C) (D) (E)

46.

Any comfortable position Left lateral decubitus Prone position Right lateral decubitus Supine position

A mother brings her 4-year-old child to the emergency department for evaluation of persistent fevers. For the last week the young girl has suffered from spiking fevers and malaise. The mother is concerned that she may have caught something while visiting family in Japan 1 month prior, though the child has no known sick contacts. Past medical history and family medical history are unremarkable. Aside from the usual childhood illnesses, the patient has been well. She is up to date on all appropriate immunizations. Physical examination shows bilateral conjunctival injection and dry, cracked mucous membranes. Her lips are fissured and her tongue is erythematous and swollen. She has marked cervical lymphadenopathy, with at least one tender node of approximately 2 cm. Additionally, across her chest is a pleomorphic, erythematous rash. Erythema is also present on the soles of her feet and palms of her hands, as is some mild desquamation. Laboratory studies show a normocytic anemia, a thrombocytosis, and a markedly elevated erythrocyte sedimentation rate. Blood and urine cultures are unrevealing. Given this patient’s presentation, which of the following is the most appropriate treatment? (A) Acetaminophen (B) Aspirin (C) Aspirin and intravenous immunoglobulin (D) Plasmapheresis (E) Prednisone

47.

A 16-year-old girl comes to the clinic for evaluation of delayed puberty. She is concerned about her development, as she feels her body has not changed a bit since she was 9 years old. Although she reports that she has always been shorter than her classmates, she has generally been healthy. She has had no major illness and knows of none that run in her family. Physical examination reveals a short, stocky appearing young woman. She has Tanner stage 1 breasts and sparse pubic and axillary hair. Her ears appear lower than usual and she has a low hairline and a small mouth and mandible. Additionally, her neck is short and she has a wide, broad chest with nipples that are farther apart than normal. She is markedly hypertensive. Which of the following is the most likely cause of this patient’s hypertension? (A) (B) (C) (D) (E)

Aortic coarctation Essential hypertension Ovarian failure Renal artery stenosis Renal failure

15

USMLE Step 2 Assessment Exam

48.

A 32-year-old multiparous woman delivers a baby boy at full term after an uneventful pregnancy. She has three healthy children at home and has no significant past medical history. She denies taking any medication during her pregnancy. The neonate scores 7 and 8 on the Apgar scale at 1 and 5 minutes, respectively. He is 48 cm long and weighs 3,200 g. His vital signs are within normal limits. On physical examination, asymmetry of the lower extremities is noted. The left leg is less well developed and the foot is slightly deformed. An x-ray of the lumbosacral spine is performed and shows laminal defects in the L5-S1 region with widening of the interpedicular spaces and butterfly vertebrae. Which of the following skin lesions is most likely to be associated with this defect? (A) (B) (C) (D) (E)

49.

A 5-year-old white boy is complaining of groin pain. He has been limping for approximately 1 month and it has gotten progressively worse over the past week. He has not had any recent infectious contacts or any known trauma. The boy is in the fifteenth percentile for height and the tenth percentile for weight. His temperature is 36.7 C (98 F). He has decreased right hip internal rotation and abduction. The patient’s knee examination is normal. When observing his gait, the physician notes that the boy leans his torso to the right side during the stance phase on the right leg. Which of the following is the most likely diagnosis? (A) (B) (C) (D) (E)

16

Ash-leaf macule Becker nevus Mongolian spot Patch of hair Salmon patch

Developmental dysplasia of the hip Juvenile rheumatoid arthritis Legg-Calve-Perthes disease Septic arthritis Slipped capital femoral epiphysis

50.

A 25-year-old woman comes to the emergency department in labor. She delivers a 7-pound baby boy without complications. She tells the obstetric staff that she has been visiting a friend for the last 2 weeks and that her medical records from her obstetrician will not be available for another 2 days. She is carrying a medical note from her doctor stating that she is hepatitis B surface antigen–positive and hepatitis B e antigen–positive. She is concerned that the baby will be affected by her illness. Which of the following is the most appropriate intervention for the newborn? (A) Administer first dose of hepatitis B vaccine only (B) Administer first dose of hepatitis B vaccine with hepatitis B immune globulin (C) Administer hepatitis B immune globulin only (D) Explain to the mother that no intervention is indicated (E) Send hepatitis serologies on the mother and treat the newborn on the basis of the results

Pediatrics Assessment Exam Answers and Explanations ANSWER KEY 1. 2. 3. 4. 5. 6. 7. 8. 9. 10. 11. 12. 13. 14. 15. 16. 17. 18. 19. 20. 21. 22. 23. 24. 25.

D B E E D C C D B D E C D E B B C B A D E F D B B

26. 27. 28. 29. 30. 31. 32. 33. 34. 35. 36. 37. 38. 39. 40. 41. 42. 43. 44. 45. 46. 47. 48. 49. 50.

E D B C C D D D A A E C D A B E C D C E C A D C B

17

USMLE Step 2 Assessment Exam

1.

2.

18

The correct answer is D. Nearly one third of children may have an innocent murmur during routine examination. Such murmurs are not associated with hemodynamic abnormalities or cardiac defects. They arise as a result of normal turbulence of blood in the chambers of the heart or large vessels during childhood. Innocent murmurs are characterized by their character (short systolic), low intensity (grade I or II), and absence of any other abnormal physical signs. No further investigation is required in such children, and parents can be reassured regarding the benign nature of the finding. A chest x-ray (choice A) is useful to evaluate heart size when cardiac failure is suspected. It also can show abnormal cardiac shape in certain congenital cardiac lesions. It is not required in the case presented in the vignette. Children with anemia have hyperdynamic circulation and can demonstrate cardiac flow murmurs. In an otherwise healthy child, however, the presence of murmur alone does not require obtaining a hematocrit (choice B). Instead, hematocrit should be performed at 1 year of age and before school entry as part of the routine health maintenance. Cardiac disease can be suspected from history of tiredness, breathlessness, or growth failure. In addition, there are features in physical examination that can help to identify children with heart murmur who have a cardiac disease. These include a pansystolic murmur, very loud murmurs (grade III or greater), presence of click, or an abnormal second heart sound. In the absence of any of these features, an electrocardiogram (choice C) or echocardiogram (choice E) is not required. The correct answer is B. This patient has erythema migrans, a manifestation of Lyme disease. Erythema migrans develops 3 to 32 days after the bite of a deer tick (Ixodes scapularis) infected by Borrelia burgdorferi. Prompt removal of the attached tick within the first 8 to 12 hours minimizes the chance of transmission of any of these diseases, but appropriate followup blood work should be done to evaluate if anti-Borrelia antibody titers are increasing and treatment with antibiotics is necessary. Commonly used antibiotics include amoxicillin, doxycycline, cefuroxime, erythromycin, and azithromycin. In the United States, the most common vector of Lyme disease is Ixodes scapularis. Intramuscular triamcinolone (choice A) is not indicated in the treatment of erythema migrans, which is a bacterial infection. Oral doxycycline (choice C) is one of the antibiotics commonly used for the treatment of erythema migrans. The age of the patient is a contraindication for the use

of a tetracycline antibiotic, however, because it may cause permanent staining of teeth if used in children younger than 8 years of age. Topical hydrocortisone (choice D) is of no benefit in the treatment of erythema migrans and should not be used, as it may mask the presence of infection through its local anti-inflammatory and vasoconstrictive properties. Topical ketoconazole (choice E) is an antifungal medication often used to treat fungal infections of the skin. Because the potassium hydroxide preparation of skin scrapings performed in this patient was negative, the described disease is not a fungal infection. 3.

The correct answer is E. Asthma is a commonly encountered problem in the emergency room. Patients can present with varying degrees of severity and require interventions based upon the initial assessment. In the child presented in this vignette, all the interventions mentioned in the choices are appropriate. However, the presence of severe distress and low oxygen saturation mandate immediate provision of inhaled oxygen to relieve hypoxia. This should be done even before a complete history can be obtained. Nebulized albuterol (choice A) is the appropriate first drug in patients with an acute attack of asthma. If relief is incomplete, the dose should be repeated immediately. The patient should be closely monitored for deterioration while inhaled therapy is being administered. Chest x-ray (choice B) is not indicated in most children with previously diagnosed asthma who present with worsening symptoms. Chest x-ray is appropriate if the acute attack is unusually severe, if pneumonia is suspected, or if breath sounds are significantly decreased on one side. Some children presenting with status asthmaticus may be dehydrated due to tachypnea and decreased intake of fluids. Thus, it is appropriate to start intravenous fluids (choice C) in such patients once the respiratory status is stabilized. Steroids (choice D) are extremely important and should be instituted early in the management of status asthmaticus. If the child has been vomiting or is in severe distress, then parenteral methylprednisolone can be used.

4.

The correct answer is E. This patient has cat-scratch disease, a self-limited infection caused by Bartonella henselae. Symptoms can vary from mild to severe and may include malaise, anorexia, fever, and headache. Three quarters of cases occur between September and March. Most patients give a history of exposure to a cat, although other small animals also may be the reservoir.

Pediatrics Answers and Explanations

widely available for detection of parvovirus B19. Detection of viral DNA in fetal blood aids in making a diagnosis of B19-induced fetal hydrops. Treatment consists of supportive care only. Coxsackievirus A16 (choice A) is the cause of handfoot-and-mouth disease, a viral illness that presents clinically with characteristic oral and distal extremity lesions. The incubation period is 3 to 6 days, after which a flu-like prodrome ensues and lasts 2 to 3 days. Oral lesions begin as erythematous macules that evolve rapidly into vesicles and then ulcerate. They are painful and interfere with eating. Cutaneous lesions in the form of asymptomatic vesicles with an oval erythematous halo are typically present on the hands, feet, and buttocks. They tend to follow skin lines and are oval in shape. Coxsackievirus infection is highly contagious. Transmission occurs by way of direct contact with nasal or oral secretions, fecal material, or aerosolized droplets in a fecal-oral or oral-oral route. Herpes simplex virus type 1 (choice B) is the cause of orolabial herpes simplex (“cold sore”). Although initial infection is usually asymptomatic, a vesicular gingivostomatitis may develop in up to 1% of people, chiefly in children and young adults. By adulthood, almost 100% of the population has antibodies confirming infection, but only approximately 10% develop recurrent episodes of orolabial herpes. Human herpesvirus type 6 (choice C) causes roseola infantum, a common cause of sudden, unexplained high fever in children 6 to 36 months of age. Prodromal fever may be accompanied by convulsions and lymphadenopathy. Suddenly, on approximately the fourth day, the fever drops and a morbilliform erythematous rash consisting of rose-colored discrete macules appears on the neck, trunk, and buttocks, and sometimes on the face and extremities. Complete resolution occurs in 1 to 2 days. Rubella virus (choice E) is the cause of rubella, a viral infection that occurs most commonly in adolescents and young adults. The patient presents with retroauricular and posterior occipital lymphadenopathy, pharyngitis, low-grade fever, and upper respiratory infection symptoms. A fleeting, erythematous, maculopapular rash begins on the face and spreads to the body, lasting up to 3 days. Forchheimer spots may be present on the soft palate, aiding in making the clinical diagnosis. The prognosis in children is excellent. If the infection occurs during pregnancy, however, it may lead to the congenital rubella syndrome.

Within 3 to 10 days, a small papule, pustule, or vesicle appears at the site of injury. A week later, edema and tenderness develop in regional lymph nodes. Untreated, the disease resolves spontaneously in 2 to 5 months except in severely immunocompromised individuals such as those with AIDS. Diagnosis is established through clinical presentation, increasing titers of antibodies, or histopathologic evaluation of tissue stained with the Warthin-Starry technique. Treatment is supportive, except in immunosuppressed individuals who require antibiotics. A nasopharyngeal swab for culture (choice A) would not help to establish the diagnosis of cat-scratch disease because the microorganisms are not present in nasopharyngeal secretions. Infection is the result of direct inoculation into the skin. Needle biopsy of the lymph nodes (choice B) is not indicated, because the diagnosis can be established based on clinical presentation and increasing serum antibody titers to Borrelia henselae. Although the diagnosis of cat-scratch disease most commonly is made based on history of contact with an animal and clinical presentation, it should be confirmed either by increasing serum antibody titers or specially stained tissue sections from biopsy of a skin lesion. It is therefore incorrect that no further studies are indicated (choice C). Radiography of the head and neck (choice D) would be noncontributory in establishing the diagnosis, as it would show only soft tissue swelling at the site of the enlarged lymph nodes. 5.

The correct answer is D. This patient has the typical clinical presentation of erythema infectiosum (or Fifth disease) caused by parvovirus B19, a DNA virus. Erythema infectiosum is a benign, self-limited exanthematous illness. Humans are the only known host and they transmit the virus by way of respiratory secretions and blood. The disease is seen most commonly in spring and has an incubation period that ranges from 4 to 28 days. The patient usually presents with mild systemic symptoms, including low-grade fever, headache, and upper respiratory tract symptoms such as pharyngitis. Some patients (mostly adults and adolescents) may have arthritic symptoms. Clinically, an intensely red “slapped cheek” appearance is followed by a lacy, reticulated maculopapular rash on the trunk and proximal extremities. The palms and soles are not affected. The rash lasts anywhere from 2 to 40 days (average, 11) but has a tendency to recur with exposure to heat or physical exertion for a period of several weeks. The patient is not contagious once the rash appears. Diagnosis of erythema infectiosum is usually made on clinical grounds, as routine laboratory studies are not

6.

The correct answer is C. Hemophilia A follows an Xlinked recessive inheritance pattern. Thus, the risk that the abnormal X-chromosome gene will be passed from

19

USMLE Step 2 Assessment Exam

the mother to the male or female offspring is 50%. However, because males possess a single X-chromosome, all the males who inherit the abnormal gene will develop hemophilia. Thus the risk of disease is 50% for male offspring. On the other hand, females possess two X-chromosomes, one from the father and one from the mother. Thus, none of the females who inherit one abnormal gene will develop the disease. Hence, the risk of disease is 0% for female offspring, not 25% (choice A). Instead, female offspring with one abnormal gene are asymptomatic carriers, like the mother. The risk of disease among male and female offspring is different in X-linked recessive inheritance (choice B), due to the presence of two X-chromosomes in females. Since fathers do not pass on the X-chromosome to male offspring, the incidence of the disease in males is not affected by paternal disease (choice D). A significant number of patients with hemophilia have a negative family history due to a high rate of spontaneous mutation in the factor VIII gene. In such cases the risk to the second male offspring cannot be ascertained without determining the mother’s carrier status. However, due to the fact that one of the maternal uncles has hemophilia A, it is reasonable to presume that the mother is a carrier for hemophilia A (choice E). This can be further confirmed by laboratory tests on the mother. 7.

The correct answer is C. This boy has a typical case of measles (morbilli, rubeola), an infectious disease caused by the Morbillivirus and transmitted through respiratory droplets. It usually occurs in unimmunized preschool children or in high school and college students. Measles is extremely contagious and up to 100% of unimmunized contacts contract the disease. Measles is associated with cough, coryza, conjunctivitis, and fever. Koplik spots are usually present on the buccal mucous membrane opposite the premolar teeth. They appear as bluish-white spots. The exanthem typically develops on or around the fourth day on the forehead and behind the ears and then spreads “like a shower” to the face, trunk, and limbs. At first it is macular, and then dull red papules form with a tendency to coalesce into irregular concentric patterns or become more diffusely confluent. From the sixth to the twentieth day the rash starts fading, leaving behind brownish discoloration and fine scaling. In very severe forms it may become hemorrhagic. Active immunization with the live attenuated vaccine (MMR, measles-mumps-rubella) has reduced the incidence of measles infection. Hepatitis B vaccine (choice A) prevents infection with the hepatitis B virus. Hepatitis B can cause acute and

20

chronic liver disease. The clinical presentation ranges from subclinical hepatitis to symptomatic hepatitis and, in rare instances, fulminant hepatitis. Long-term complications of hepatitis B include cirrhosis and hepatocellular carcinoma. Perinatal or childhood infection is associated with few or no symptoms, but it has a high risk of becoming chronic. Influenza vaccine (choice B) is indicated in patients with chronic lung disease, heart disease, immunosuppression, and hemoglobinopathies, and in healthcare workers and patients in nursing homes and long-term care facilities. It is not routinely administered to healthy children. Influenza presents with similar prodromal symptoms as measles, but photophobia, Koplik spots, and skin exanthems are not seen in influenza. Meningococcal vaccine (choice D) is not a routine school vaccine, although it is recommended or required by some colleges. Indications include anatomic or functional asplenia, patients with complement deficiencies, and travelers to endemic areas. Meningococcal infection presents with meningeal symptoms (e.g., headache, nuchal rigidity, and photophobia) and an increased number of white blood cells in the cerebrospinal fluid (CSF), i.e., pleocytosis. Varicella vaccine (choice E) is scheduled for all children 12 to 18 months of age and all children who have no history of varicella. The prodrome of fever, headache, malaise, and coryza is followed by a typical intensely pruritic vesicular rash that appears in crops. 8.

The correct response is D. The term premature adrenarche (or pubarche) refers to growth of pubic hair before the age of 8 years in girls (9 years in boys) without the appearance of any other signs of sexual maturation. Sparse hair grows on labia majora and the mons pubis in girls. The girls generally have height and osseous maturation that are more advanced than their peers. Premature pubarche is a benign condition and does not portend the presence of a serious underlying illness. However, in those cases where pubic hair is accompanied by clitoromegaly or other evidence of masculinization, further endocrine investigation is warranted. Variants of congenital adrenal hyperplasia (also known as nonclassical forms) can present with virilization in young girls (choice A). However, other evidence of masculinization—such as clitoromegaly, change in voice, or growth acceleration—are generally present. Exogenous androgens—for example, in the form of anabolic steroids—can produce hirsutism and clitoromegaly (choice B). However, there is no history of ingestion of androgens in the present case.

Pediatrics Answers and Explanations

Onset of secondary sexual characteristics in girls before the age of 8 years is considered to be precocious puberty, and not normal pubertal development (choice C). Girls with Turner syndrome (45,XO) have delayed sexual development (choice E). They can present in the teenage years with pubic and axillary hair without breast development. However, short stature is an important hallmark of this syndrome, unlike the case vignette. 9.

The correct answer is B. This girl has the typical presentation of roseola infantum (exanthema subitum), a febrile illness that usually occurs in children up to 5 years of age. Human herpesvirus 6 is responsible for this infection, and the incubation period is 5 to 15 days. Infection usually occurs early in life, with peak incidence at 6 to 15 months. The classic presentation is high fever up to 41.0 C (106.0 F) that lasts 3 to 4 days with minimal physical findings that may include rhinorrhea, conjunctivitis, and mild occipital lymphadenopathy. Suddenly, on approximately the fourth day, the fever decreases and a morbilliform erythema consisting of rose-colored discrete macules appears on the neck, trunk, and buttocks, and sometimes on the face and extremities. The mucous membranes are spared. Complete resolution occurs in 1 to 2 days. No diagnostic studies are necessary. The diagnosis is based on the age of the patient, history, and physical findings. Supportive therapy with antipyretics and fluids is sufficient. Measles (choice A) is an infectious disease caused by the Morbillivirus and is transmitted through respiratory droplets. Measles is associated with cough, coryza, conjunctivitis, and fever. Koplik spots are usually present on the buccal mucous membrane opposite the premolar teeth. They appear as bluish-white spots. The exanthem typically develops on or around the fourth day on the forehead and behind the ears and then spreads “like a shower” to the face, trunk, and limbs. At first it is macular, and then dull red papules form with a tendency to coalesce into irregular concentric patterns or become more diffusely confluent. From the sixth to the twentieth day the rash starts fading, leaving behind brownish discoloration and fine scaling. Rubella (choice C) is a viral infection caused by the rubella virus that occurs most commonly in adolescents and young adults. The patient presents with retroauricular and posterior occipital lymphadenopathy, pharyngitis, low-grade fever, and upper respiratory infection symptoms. A fleeting, erythematous, maculopapular rash begins on the face and spreads on the body, lasting up to 3 days. Forchheimer spots may be present on the soft palate, aiding in making the clinical diagnosis. The prognosis in children is excellent. If the infection occurs

during pregnancy, however, it may lead to the congenital rubella syndrome. Scarlet fever (choice D) is the result of pyrogenic exotoxins produced by beta hemolytic streptococcus group A. Fever and sore throat precede the skin eruption by several days. The rash begins on the face and neck and spreads downward, sparing palms, soles, and the perioral area (circumoral pallor). Tiny scaly papules appear on the trunk, giving the skin the texture of sandpaper. Fine desquamation ensues from the face and neck down, whereas palms and soles often desquamate in a “gloves and socks” fashion. The erythema is accentuated in skin folds, where petechiae may also occur. The most characteristic mucosal lesion is the strawberry tongue: protruding, enlarged, erythematous papillae and a thick white coating. A diagnosis of scarlet fever may be made clinically and is supported by throat cultures or rapid strep testing. Varicella (choice E), or chickenpox, is caused by the varicella zoster virus. Children between the ages of 5 and 9 years are affected most commonly. Clinically, chickenpox presents as a rash, low-grade fever, and malaise. In the immunocompetent patient, this is usually a benign illness that is associated with lassitude and body temperatures of 37.8 to 39.4 C (100.0 to 103.0 F) of 3 to 5 days’ duration. The skin lesions include maculopapules, vesicles, and scabs in various stages of evolution. These lesions, which evolve from maculopapules to vesicles over hours to days, appear on the trunk and face and rapidly spread to involve other areas of the body. Successive crops appear over a 2- to 4-day period. Lesions can be found on the mucosa of the pharynx and the vagina also. Immunocompromised individuals are at greater risk for visceral complications, which occur in 30 to 50% of cases and are fatal 15% of the time. 10.

The correct answer is D. This patient has neurofibromatosis, an autosomal dominant disorder that occurs as a result of an abnormality of neural crest differentiation during embryogenesis. The diagnosis of type 1 neurofibromatosis requires two or more of the following criteria: 1) six or more café-au-lait macules of more than 5 mm in greatest diameter in prepubertal individuals and more than 15 mm in diameter in postpubertal individuals; 2) two or more neurofibromas of any type or one plexiform neurofibroma; 3) freckling in the axillary and/or inguinal region; 4) optic glioma; 5) two or more Lisch nodules;

21

USMLE Step 2 Assessment Exam

the urinary tract, which might have been produced by vesicoureteral reflux. The reflux itself, however, is best demonstrated with the voiding cystourethrogram. Because it is so safe, ultrasound often is used to begin urologic workups in children. Had the question asked for the standard next step in management, ultrasound might have been a reasonable answer. Retrograde urethrogram (choice D) is used to evaluate urethral injuries in male patients with pelvic fracture and blood at the meatus.

6) a distinctive osseous lesion, such as sphenoid dysplasia or thinning of the long-bone cortex with or without pseudoarthrosis; and 7) a first-degree relative (parent, sibling, or offspring) with the disease. To make the diagnosis of neurofibromatosis type 2, one of the following is needed: 1) bilateral eighth nerve masses (acoustic neuroma); or 2) a parent, sibling, or child with neurofibromatosis type 2. There is no specific therapy for neurofibromatosis. Genetic counseling should be done and prenatal testing is available. Therapy should include early identification and treatment of complications. A pediatric ophthalmologist should perform an annual ophthalmology examination. Jaw cysts (choice A) are typically seen in the basal cell nevus (Gorlin) syndrome, in association with multiple basal cell carcinomas of the skin and pitting of the palmoplantar surfaces. Osteopoikilosis (choice B) is characteristic of Goltz syndrome, or focal dermal hypoplasia, an X-linked dominant disorder that affects only females. Linear and whorled, Blaschko line–associated papules and plaques with fat herniation are noted on the skin. Mild to moderate mental retardation is typical and patients may have an associated convulsive disorder. Osteopathia striata (choice C) is a manifestation of Buschke-Ollendorf syndrome, where connective tissue nevi are the hallmark of skin involvement. “Train track” calcifications of the skull (choice E) were historically used in the diagnosis of Sturge-Weber syndrome, a neurophakomatosis characterized by port wine stains of the V1 (±V2, V3) trigeminal area of the face, leptomeningeal vascular anomalies with or without associated seizure disorders, and possible ipsilateral glaucoma. 11.

22

The correct answer is E. The clinical suspicion in this case is vesicoureteral reflux, which is best demonstrated by filling the bladder with dye and doing fluoroscopy while the child voids. Cystoscopy (choice A) as a rule is not indicated for the workup of suspected reflux. It might be needed to further define abnormal anatomy, for instance, an ectopic ureter. It is not, however, the best initial choice for this question. Intravenous pyelogram (choice B) or ultrasound (choice C) can demonstrate the presence of dilation of

12.

The correct answer is C. This patient has rubella (German measles), a viral infection characterized by a maculopapular exanthem and tender lymphadenopathy of the posterior occipital, retroauricular, and cervical lymph nodes. It is seen infrequently because of widespread vaccination efforts. The incubation period is 14 to 21 days. Patients are contagious 2 days before the rash begins and 5 to 7 days after the rash. Clinically, pharyngitis, low-grade fever, and upper respiratory tract infection symptoms may be the presenting sign or rubella may begin with tender lymphadenopathy. The exanthem begins on the face and rapidly spreads to the entire body in the form of erythematous macules and papules that resolve in 3 days. Retroauricular, posterior cervical, and postoccipital lymph nodes are characteristically enlarged and slightly tender. Forchheimer spots are rose spots seen on the soft palate that may precede the skin rash. Adolescent females may have polyarthritis. The diagnosis usually is made clinically from the history and physical examination but should be confirmed by serologic tests or a viral culture. Supportive treatment is given. The prognosis of rubella in children is excellent and complications are uncommon. A pregnant woman who contracts rubella in the first trimester of pregnancy, however, has a high probability that intrauterine transmission will give rise to the congenital rubella syndrome. His 4-month-old niece (choice A) is highly unlikely to contract rubella because of her very young age. Rubella most commonly affects adolescents and young adults. His 15-year-old sibling (choice B), although likely to contract rubella because of his age, has an excellent prognosis and complications would be uncommon. His 60-year-old grandmother with systemic lupus erythematosus (choice D) and his 75-year-old grandfather with diabetes mellitus (choice E) are highly unlikely to contract rubella because of their age. The accompanying systemic diseases (i.e., diabetes mellitus and systemic lupus erythematosus) do not carry an increased risk for infection with this virus, nor are they a known risk factor for complications.

Pediatrics Answers and Explanations

13.

14.

The correct answer is D. Osgood-Schlatter disease is a traction apophysitis of the tibial tubercle near the insertion of the patellar tendon. The condition is most common in boys during late childhood or adolescence, especially in athletes. Diagnosis is based on characteristic presentation and examination findings. An x-ray is useful to exclude other conditions. The only treatment required is a restriction or decrease in physical activity. Complete resolution occurs when tibial tubercle fuses to the tibia as part of bone maturation, usually over the following 1 to 2 years. Therefore, reassurance that the condition is self-limiting is appropriate. Acute leukemia can occasionally present with bone pain in children. The presence of systemic symptoms, organomegaly, lymphadenopathy, bruising, or pallor would make it necessary to obtain a complete blood count (choice A) in a child presenting with bone or joint symptoms. Erythrocyte sedimentation rate (choice B) is elevated in most rheumatic conditions. The absence of systemic symptoms and lack of physical findings affecting the knee joint in this patient make the diagnosis of a rheumatic disorder less likely. In the presence of typical physical signs, magnetic resonance imaging (choice C) is not indicated, and referral to an orthopedic surgeon (choice E) is not necessary. The correct answer is E. Oral thrush is common in the newborn period and arises from infection of the mucous membrane by Candida. Infants become colonized by acquiring the infection from the mother during delivery. The use of antibiotics can lead to development of thrush. Other individuals who are susceptible are those with immunodeficiency (congenital or acquired) and diabetes mellitus. The whitish plaques of thrush can be confused with milk curds in newborns, but the latter can be easily removed with a spatula and do not result in punctate bleeding spots. Treatment is usually with nystatin suspension that acts locally on the oral mucosa. Clotrimazole and miconazole are also effective therapies. Amoxicillin (choice A) is an antibiotic commonly used to treat respiratory infections in children. It has no antifungal activity. Amphotericin B (choice B) is an antifungal agent that is used parenterally to treat serious systemic fungal infections. It has no role in treating infections confined to the skin or oral mucosa. The frequency of oral thrush is no different in formulafed infants, and change to formula feeding (choice C) is not indicated.

Although sometimes very mild cases of thrush may not require treatment (choice D), a newborn who is fussy or has feeding problems should be appropriately treated. 15.

The correct answer is B. Infectious croup, or laryngotracheobronchitis (LTB), is an acute, self-limited illness in children under the age of 3 years. The most frequent etiology of LTB is parainfluenza virus. The illness resolves in 3 to 7 days and usually does not require hospitalization. Diagnosis is based on the clinical picture, or sometimes on x-ray of the airway, which shows characteristic narrowing of the subglottic area described as pencil-point sign or steeple sign. No further studies are required to make the diagnosis. Haemophilus influenzae type B (choice A) can cause epiglottitis, a severe and potentially fatal illness in children between 3 and 5 years of age. The patients present with high fever, rapidly progressive airway obstruction, and toxic look. This illness has become rare due to immunization. Respiratory syncytial virus (choice C) causes bronchiolitis and pneumonia in infants. It is a much less frequent cause of LTB than parainfluenza virus. Group A streptococcal infection (choice D) presents with a sore throat in older children, often with tonsillar exudates, palatal petechiae, and enlarged, tender, cervical lymph nodes. Stridor is unlikely in streptococcal infections. Streptococcus pneumoniae (choice E) causes otitis media and pneumonia in children. It does not cause croup or inspiratory stridor.

16.

The correct answer is B. The two most common causes of microcytic anemia when routine testing is done at the age of 1 year are iron deficiency and thalassemia trait. Between the two, iron deficiency is far more prevalent. In children who have mild anemia (hemoglobin 9.5 to 11 g/dL) in screening complete blood count or hematocrit at 1 year of age, it is appropriate to give a trial of oral iron at a dose of 3 mg/kg/day for 1 to 2 months. If there is no response in the hemoglobin at this time, then iron therapy should be not be continued without further investigations. These should include complete blood count, reticulocyte count, serum iron level, serum ferritin level, and iron-binding capacity. Supportive evidence of insufficient iron in the diet should also be sought; most often there is history of excessive milk intake or use of non–iron-containing infant formula. In patients with thalassemia trait (either alpha- or beta-thalassemia), the blood count looks similar to iron deficiency due to the presence of low hemoglobin and low mean corpuscular volume (microcytosis). In thalassemia-trait, the red cell

23

USMLE Step 2 Assessment Exam

frothy leukorrhea. In men, it may cause urethritis, prostatitis, or balanoposthitis (inflammation of the glans and prepuce), but is very frequently asymptomatic. Chains of Gram-negative bacilli (choice B) are found in 50% of smears performed in patients with chancroid, a sexually transmitted infection with Haemophilus ducreyi. Clinically, one or more deep or superficial tender ulcers on the genitals are accompanied by painful inguinal adenitis in half of the cases. Motile spiral spirochetes on darkfield microscopy (choice D) are diagnostic of syphilis, a sexually transmitted infection with Treponema pallidum. Clinically, the primary lesion is an asymptomatic small red papule or crusted erosion that indurates and assumes a cartilaginous-like consistency over several days. The surface of the erosion typically oozes a serous fluid. Regional lymph nodes are enlarged, nontender, and firm. Multinucleated giant cells are seen in Tzanck smears (choice E) (methylene blue staining) of genital herpes simplex lesions. Clinically, painful grouped vesicles may be accompanied by tender adenitis. Genital herpes is most commonly caused by herpes simplex virus type 2, although contemporary sexual practices have caused an increase in the number of cases caused by herpes simplex virus type 1.

distribution width (RDW) remains normal, whereas it is elevated in iron deficiency. Such patients do not respond to oral iron supplements. The child in this vignette has microcytic anemia with a normal RDW and did not respond to ferrous sulfate drops. In addition, his diet seems to contain adequate amounts of iron. Thus he may have thalassemia trait, for which the most informative test is hemoglobin electrophoresis. Bone-marrow aspiration (choice A) is appropriate when aplastic anemia or leukemia is suspected. However, it is not required in this case. Serum B12 level (choice C) is useful in diagnosis of megaloblastic anemia, which manifests as increased MCV. It is not needed in patients who have microcytosis. Serum iron and ferritin level (choice D) should be obtained in this child to rule out iron deficiency that has not responded to oral iron therapy. However, these tests will not likely reveal the etiology of the anemia. Upper and lower gastrointestinal endoscopy (choice E) are important tests where iron deficiency is suspected to be due to blood loss from the gastrointestinal tract. It is not required for the child presented in this vignette. 17.

24

The correct answer is C. This patient has the classic presentation of gonorrhea. Gonorrhea is a sexually transmitted disease caused by Neisseria gonorrhoeae. It affects the mucous membranes of the genitourinary tract and may at times also affect the mucosa of the oropharyngeum, rectum, and conjunctiva. Clinical presentation may vary. Some patients will have urethritis, cervicitis, or dysuria. Other patients with gonorrhea may be asymptomatic. Boys with urethritis will have purulent discharge and burning on urination. Postpubertal girls with symptomatic cervicitis may have purulent discharge, suprapubic pain, and dysuria. The cervix may be inflamed and friable. A culture for gonorrhea should be performed on any discharge from the cervix, urethra, rectum, or eye. If disseminated gonorrhea is suspected, cultures of the blood, pharynx, rectum, urethra, cervix, and synovial fluid (if applicable) should be evaluated. The discharge should also be Gram-stained. A Gram stain indicative of gonorrhea will show Gram-negative intracellular diplococci in polymorphonuclear cells. All patients with gonorrhea should be tested for syphilis and HIV. Ceftriaxone is the treatment of choice. In addition, doxycycline or azithromycin should be administered to treat presumed or proven Chlamydia. Colorless pyriform flagellates in a saline smear (choice A) are diagnostic of trichomoniasis, a sexually transmitted infection with Trichomonas vaginalis. It is a common cause of vaginal pruritus, burning, and a

18.

The correct answer is B. The obvious diagnosis is congenital diaphragmatic hernia, but the emergent issue is the pulmonary hypoplasia on the left and pulmonary hypertension that affects both lungs. Respiratory support is required for a few days before surgical correction is attempted. As suggested, the infant is not ready to undergo surgical correction yet (choice A). A chest tube (choice C) might be needed if positive pressure ventilation were to produce a pneumothorax, but right now that is not what the infant needs. There is high pulmonary vascular resistance now, and that is part of the problem. To increase it further (choice D) would make things worse. Drugs with the opposite effect (such as nitric oxide) may be used. The infant needs positive pressure ventilation, but if it is done through a mask (choice E), part of that gas will end up in the gastrointestinal tract, further compressing the lungs.

19.

The correct answer is A. Charcot-Marie-Tooth disease (hereditary motor-sensor neuropathy type I) is the most common genetic neuropathy. In this autosomal dominant disease, patients have peroneal muscular atrophy. Most patients are asymptomatic until late childhood or early adolescence. Patients have a history of gait

Pediatrics Answers and Explanations

disturbance, clumsiness, and tripping over their own feet. By the teenage years, patients exhibit pes cavus, tremor, and variable sensory loss (stocking-glove distribution). On physical examination, the peroneal and tibial nerves are most commonly affected. There is muscle wasting of the lower legs, giving them a stork-like appearance. Patients have claw hands with nerves that are palpably enlarged. There is decreased motor and sensory conduction. A sural nerve biopsy demonstrates “onion bulb” formations that surround the axons. This pathologic finding is called interstitial hypertrophic neuropathy. The definitive genetic diagnosis is made with genetic testing of the blood. Orthotics are used to brace the feet and the legs. Parents should be examined and nerve conduction studies performed. Duchenne muscular dystrophy (choice B) is the most common hereditary neuromuscular disease. It is transmitted as an X-linked recessive disease, with approximately 30% new mutations. Clinically, poor head control may be one of the first signs of the disease. There could be mild delay in early gross motor skills. On physical examination, patients will have pseudohypertrophy of the calves. Gower sign is seen by 3 years of age. Hip girdle weakness and a Trendelenburg sign (at 5 to 6 years of age) develop. Patients eventually lose ambulation capabilities. Patients have a poor cough and pharyngeal weakness. Death usually occurs by 18 years of age from respiratory problems and/or, rarely, heart problems. Friedrich ataxia (choice C) is a hereditary metabolic degenerative disease caused by an expanded GAA triplet repeat in the frataxin gene (9q13-q21.1). It is transmitted as an autosomal recessive or dominant trait. Ataxia usually appears before 10 years of age. Cardinal features include progressive limb and gait ataxia, dysarthria, loss of joint position and vibration senses, absent tendon reflexes in the legs, and extensor plantar responses. These patients have an explosive dysarthric speech, and nystagmus. Titubation is present when sitting or standing. The diagnosis is based on clinical signs and symptoms. There is no available treatment. This is a progressive disorder with significant morbidity. Loss of ambulation typically occurs 15 years after disease onset. Guillain-Barré syndrome (choice D) is a postinfectious polyneuropathy that causes demyelination in motor and sometimes sensory nerves. This disease affects patients of all ages. Campylobacter jejuni and Mycoplasma pneumoniae have been associated with Guillain-Barré. Clinically, an ascending weakness and paralysis that begins in the lower extremities can be seen. This usually occurs 10 days after a nonspecific viral infection. Weakness may progress to include the

respiratory muscles, causing respiratory insufficiency. In most cases, there is a spontaneous recovery in 2 to 3 weeks. Involvement of the respiratory muscles may result in respiratory failure. Werdnig-Hoffman disease (choice E) is a degenerative disorder of motor neurons. The disease is transmitted as an autosomal recessive trait. Onset is before 2 years of age and often begins in utero. Patients have generalized weakness and severe hypotonia of the proximal and distal limbs, intercostals, and bulbar muscles. The legs tend to lie in a frog-leg position with hips abducted and knees flexed. Fasciculations are visible in the tongue, and the patient has flaccid quadriplegia. There is no medical treatment to stop progression of the disease. Therefore, only supportive care is given. Most patients die before the age of 2 from respiratory failure and food aspiration. 20.

The correct answer is D. This patient has genital herpes, a sexually transmitted infection caused by herpes simplex virus (HSV). HSV is a double-stranded, DNA-containing, enveloped virus that causes a number of problems involving the eye, skin, oral mucosa, central nervous system, and genital tract. There are two types of HSV, type 1 and type 2. HSV-1 causes infections of the mouth, lips, eyes, central nervous system, and, uncommonly, genital disease. HSV2 is responsible for genital infections and neonatal infections and may cause oral lesions. Genital herpes is more commonly seen in teenagers and adults. It is transmitted by sexual activity. Fever, regional adenopathy, and dysuria may be present in patients with primary genital herpes infection. In girls, the vulva and vagina may have vesicles and ulcers, but the cervix is the primary site of infection. In boys, vesicles and ulcers may be found on the penis. Diagnosis may be confirmed by the Tzanck test on scrapings from the herpetic lesions. An early vesicle, not a pustule or crusted lesion, is unroofed, and the base of the lesion is scraped gently with a scalpel blade. The material is placed on a glass slide, air-dried, and stained with Giemsa or Wright stain. Multinucleated giant cells with intranuclear inclusions suggest the presence of herpes, but culture or immunofluorescence testing must be performed to identify the specific virus. ELISA and immunofluorescent techniques may be helpful. The diagnosis may also be made clinically. Acyclovir is the treatment for herpes. Valacyclovir, a prodrug of acyclovir and famciclovir, and a prodrug of penciclovir are other oral antiherpes drugs used to treat genital herpes. Genital herpes is a risk factor for human immunodeficiency virus infection. Genital herpes may be present without symptoms. During this latent infection, herpes virus is being shed and sexual partners are

25

USMLE Step 2 Assessment Exam

at risk. If the female patient with recurrent genital herpes is pregnant and has active lesions, then the infant passing through the birth canal is at risk for acquiring infection. The clinical picture of this patient is typical for primary genital herpes (fever, regional adenopathy, and vesicles and ulcers on the genitals). It would be reasonable to perform a viral culture and typing to determine the strain of virus that caused the infection, but performing a bacterial culture (choice A) is not indicated because there are no signs of bacterial infection. A complete blood count (choice B) would not aid in the diagnosis of primary genital herpes. A KOH preparation (choice C) is performed on scaling skin lesions when a fungal etiology is suspected. The edge of such a lesion is scraped gently with a scalpel blade; the removed scale is collected on a glass microscope slide and treated with 1 to 2 drops of a solution of 10 to 20% KOH. KOH dissolves keratin and allows easier visualization of fungal elements. Brief heating of the slide accelerates dissolution of keratin. When the preparation is viewed under the microscope, the refractile hyphae will be seen more easily when the light intensity is reduced. This technique can be used to identify hyphae in dermatophyte infections, pseudohyphae and budding yeast in Candida infections, and fragmented hyphae and spores in tinea versicolor. Even though a skin biopsy (choice E) is a minor surgical procedure, it is not necessary to perform an invasive procedure that might result in a permanent scar in a straightforward case that can be easily confirmed with a simple Tzanck preparation from the base of a lesion. 21.

26

The correct answer is E. It is the responsibility of the treating physician to report any suspected child abuse to the appropriate agency immediately. It is also the law in most states. All states have an identified agency responsible for the investigation of all such cases. The agency is required to make a determination about the most suitable placement for the child during the investigation and thereafter. In many states, this agency is called the Department of Child and Family Services. The physician may pursue any diagnostic investigations or treatment considered necessary for the child’s care, even if parental consent is not available (choice A). It is not appropriate for the physician to confront the baby-sitter (choice B) or any other person involved in the child’s care. What is appropriate is for the physician to obtain a thorough history in a nonconfrontational and nonjudgmental manner. The physician’s role is to gather all of the medical facts. It is the role of the state

agency or police to question potential suspects and conduct the remainder of the investigation. Many hospitals have an identified department or person who can facilitate referrals for suspected child abuse. Often this individual is a social worker (choice C). Typically, there is a social worker on call for a hospital 24 hours a day to handle such situations. It is not appropriate to wait until the following morning to contact a social worker. In cases of suspected child abuse, the physician’s duty to protect the child against further injury overrides any consideration of confidentiality (choice D). 22.

The correct answer is F. Neonates exhibit several innate responses that appear to promote infant-maternal interaction. A neonate will orient its head toward a human voice and learn to respond preferentially toward its own mother’s voice. A neonate will also fix its eyes on the human face, and preferentially fixates on a familiar face such as its mother’s. The grasp reflex (choice A) is elicited by tactile stimulation of the midpalm and leads to flexion of the digits. It allows the newborn infant to grasp its mother, just as an arboreal primate would. The grasp reflex disappears at age 3 months. The rooting reflex (choice B) is elicited by lightly stimulating the infant’s cheek and observing the reflexive attempts to bring the stimulating object (such as a nipple) to the mouth. The asymmetric tonic neck reflex (choice C) is an early reflex pattern that appears at age 2 to 4 weeks and disappears at age 6 months. This reflex is not seen immediately after birth because the newborn has high flexor tone throughout the body. The normal newborn cries immediately following a painful stimulus. A latency period (choice D) is not normal and may be present in an infant who is neurologically depressed or impaired. A newborn infant is unable to roll over from back to stomach (choice E). This milestone is usually achieved at age 4 to 8 months.

23.

The correct answer is D. The combination of delirium, mydriasis, dryness of skin and mucous membranes, tachycardia, and fever suggest cholinergic (muscarinic) blockade that most commonly would be caused by ingestion of atropine-like drugs. The drug of choice for therapy, and as a diagnostic aid, is physostigmine. Physostigmine is a potent peripheral and central anticholinesterase. Like many other anticholinesterases, it penetrates the blood-brain barrier to counteract the

Pediatrics Answers and Explanations

central effects of the cholinergic blocking agents. It must be administered cautiously and the patient monitored in an intensive care unit because of the tendency of this drug to cause cardiac arrhythmias and hypotension. Edrophonium (choice A) would have muscarinic and cholinergic effects peripherally, but not centrally. Furthermore, its duration of action is for a very short period of time. Phenobarbital (choice B) would further impair the patient’s sensorium. A preferable anticonvulsant would be diazepam. Phenothiazine (choice C) has cholinergic blocking action and would only worsen the patient’s condition. Thiamine (choice E) would be ineffective, although not harmful. 24.

The correct answer is B. The greatest risk of foreignbody aspiration is among toddlers. Small toys (or parts of toys), seeds, and nuts are among the objects most frequently implicated in such incidents. Unfortunately, a history of choking is not always elicited and the diagnosis should always be suspected when evaluating unexplained respiratory symptoms in this age group. Localized wheezing and unequal breath sounds are important diagnostic clues during examination. The diagnosis is confirmed in most cases by chest x-ray, which shows either collapse or hyperinflation (due to air-trapping) of the affected lung. Bronchoscopy is diagnostic in doubtful cases and the foreign body can be removed at the same time. Some patients with chronic cough may have aberrant blood vessels or congenital anomalies that compress the bronchus and may be visible on barium swallow (choice A). Gastroesophageal reflux can also be detected by barium swallow, as can an esophageal pH probe study (choice C). Such patients often become symptomatic during infancy. Asthma is the most common etiology of recurrent cough in children. Children often have longer duration of symptoms and wheezing at the time of presentation. In this case, the etiology of symptoms could be asthma, however the absence of previous symptoms and the localized signs on lung examination make it important to rule out foreign body in this age group. If asthma is suspected, a trial of inhaled bronchodilators should be given as a therapeutic trial rather than performing skin allergen testing (choice D). Cystic fibrosis is always possible in children with chronic respiratory symptoms (choice E). In this child, the absence of gastrointestinal symptoms, short duration of cough, and normal growth would make cystic fibrosis an unlikely diagnosis and a sweat chloride test unnecessary.

25.

The correct answer is B. This patient has the typical clinical features of spinal muscular atrophy type I (infantile form), or Werdnig–Hoffman disease, a degenerative disorder of motor neurons. The primary pathology is atrophy of anterior horn cells in the spinal cord and of motor nuclei in the brain stem, with secondary atrophy of motor nerve roots and of muscle. The disease is transmitted as an autosomal recessive trait. Onset is before 2 years of age and often begins in utero. Patients have generalized weakness and severe hypotonia of the proximal and distal limbs, intercostals, and bulbar muscles. The legs tend to lie in a frog-leg position, with hips abducted and knees flexed. Fasciculations are visible on the tongue, and the patient has flaccid quadriplegia. Tendon stretch reflexes are absent. Infants have normal intelligence. The electromyelogram shows fibrillation and evidence of muscle denervation. The serum enzyme determinations (creatine kinase) are normal or slightly elevated. Muscle biopsy shows evidence of denervation, and nerve biopsy shows slowed conduction. There is no medical treatment to stop progression of the disease. Therefore, only supportive care is given. Most patients die before the age of 2 from respiratory failure and food aspiration. Patients with Werdnig-Hoffman disease have normal intelligence, facial movements, sensation, and sphincter function. The disease is not associated with severe mental retardation (choice A). Werdnig-Hoffman disease is inherited as an autosomal recessive gene defect located on chromosome 5. The defective gene is a neuronal apoptotic inhibitor protein. The gene mutation is, hence, not located on the Xchromosome (choice C). Serum creatine kinase levels are either normal or only slightly elevated in this disease. They are not the diagnostic test of choice (choice D). The diagnosis is made by demonstrating atrophy of the anterior horn cells on autopsy or by identification of mutations in the neuronal apoptotic inhibitor protein gene in the affected patient. It is incorrect to say that this child will never attain normal sphincter function (choice E); this disease does not affect the sphincter muscles, and function there is normal.

26.

The correct answer is E. Trisomy 21, or Down syndrome, occurs in 1 in 600 live births and is the most common trisomy syndrome. About 95% of the cases are due to trisomy of chromosome 21, and the remaining 5% are due to translocations involving chromosome 21 and another chromosome. Cardiac defects are seen in nearly 40% of patients, with atrioventricular canal defect being a typical cardiac malformation. Gastrointestinal malformations (particularly duodenal

27

USMLE Step 2 Assessment Exam

atresia) are also seen in 5% of cases. Other problems include variable degrees of mental retardation, characteristic facial appearance (described in the case vignette), and hypotonia. The risk of leukemia is increased in patients with Down syndrome. DiGeorge syndrome (choice A) is due to abnormalities of chromosome 22, and its features include aortic stenosis, parathyroid and thymic aplasia, ear defects, and cleft palate. In Prader-Willi syndrome (choice B) there is severe hypotonia and failure to thrive during infancy. Older children are obese and have developmental delay. Both trisomy 13 and 18 (choices C and D) are rare and have more severe malformations than trisomy 21, with most patients dying during infancy. In trisomy 13, cleft lip and palate, heart defects, omphalocele, and polydactyly are present. In trisomy 18, the distinguishing features are microcephaly, cardiac defects, overlapping fingers, and prominent calcaneus. 27.

28

The correct answer is D. Bronchiolitis is one of the most common respiratory illnesses in infants during the winter season. It is most commonly due to respiratory syncytial virus (RSV), which causes inflammation of the small airways, causing obstruction and trapping of air. Clinically, this manifests as coughing, wheezing, and hyperinflated lung fields on chest x-ray. Often there is a mild fever and a prominent rhinitis, which can increase the breathing difficulty by causing nasal blockage. The diagnosis, based upon demonstration of RSV antigen in the nasopharyngeal aspirate, is both rapid and specific. Bronchoscopy (choice A) is very important when foreign-body aspiration is suspected. Toddlers form the highest risk group for aspiration of foreign body, which is uncommon in very young infants. Chest x-ray (choice B) will show hyperinflation of lung fields or sometimes interstitial infiltrates in infants with bronchiolitis. The changes are not specific for RSV infection. Complete blood count (choice C) is helpful for diagnosis of bacterial pneumonia (leukocytosis with increased neutrophils) and pertussis (lymphocytosis). It is generally not helpful in diagnosing RSV infection. Throat swab (choice E) is used when acute pharyngitis is suspected. This is usually a problem in older children who present with sore throat and tonsillar enlargement with exudates. Rapid diagnosis of streptococcal infection is possible on throat swabs obtained at the office.

28.

The correct answer is B. Retropharyngeal abscesses can develop in preschool children due to the presence of lymph nodes behind the posterior pharyngeal wall. It is usually a complication of acute pharyngitis, although some cases arise as a result of penetrating trauma or osteomyelitis affecting the cervical vertebrae. Group A Streptococcus, Staphylococcus aureus, and anaerobes are the most common causative agents. There is sudden onset of high fever and the enlarging abscess causes pain and obstruction, which prevent the child from swallowing saliva. The neck is held in hyperextension and the breathing can become difficult. A lateral neck xray will show increase in retropharyngeal soft tissue and help to confirm the diagnosis. CT of the neck can help in cases where the diagnosis is in doubt. A complete blood count (choice A) is likely to show leukocytosis with increased neutrophils in such patients, but is a nonspecific finding. The pain can cause patients with retropharyngeal abscess to hold their necks in rigid extension, thus raising the possibility of meningitis (choice C). However, the presence of upper airway obstruction and normal sensorium should make this diagnosis less likely and so a lumbar puncture would not be helpful in establishing a diagnosis. Nasopharyngeal aspirate (choice D) can be cultured to diagnose viral infections. Retropharyngeal abscess is a bacterial illness, and thus this test is not helpful in this patient. Throat swab (choice E) is useful for diagnosis of bacterial pharyngitis, but does not help in establishing a diagnosis of retropharyngeal abscess.

29.

The correct answer is C. Erythema toxicum is a common condition in the neonatal period, manifesting as a transient rash of unknown etiology. The diagnosis is made on characteristic visual appearance of vesicles and erythema. However, if the vesicular fluid were observed under a microscope it would reveal eosinophils. The rash disappears within a few days without specific treatment. Allergy to cow’s milk protein (choice A) is uncommon and does not present this early. More often, there are prominent gastrointestinal symptoms and blood loss in such infants. Atopic dermatitis (choice B) is a common skin disorder among infants that begins between 1 and 6 months of age, with red, itchy papules most prominent over the cheeks.

Pediatrics Answers and Explanations

Abdominal ultrasound (choice A) will reveal absence of normal ovarian structure and the presence of any renal anomalies. However, these features are not sufficient to confirm a diagnosis of Turner syndrome. Gonadotropin level (choice B) in the serum is elevated in the presence of primary ovarian failure. When the failure of puberty is due to hypothalamic causes, serum gonadotropin levels are suppressed. Patients with congenital growth hormone deficiency manifest severe growth failure with normal body proportions (choice C). The growth pattern and physical features in the present case more strongly favor a diagnosis of Turner syndrome. Acquired hypothyroidism (choice E) leads to marked deceleration of growth after an initial period of normal growth. Girls with Turner syndrome have increased incidence of thyroid abnormalities.

Intrauterine infections (choice D) can cause several forms of rash in newborns, but the most common presentation is with petechial rash. The infants tend to be small for gestational age and have organomegaly, microcephaly, chorioretinitis, or other manifestations to support the diagnosis. Sepsis (choice E) can produce a rash due to coagulopathy and thrombocytopenia. However, such infants look sick and are unable to continue with normal feeding. Temperature instability, especially hypothermia, can be observed in newborns with sepsis. 30.

31.

The correct answer is C. While less frequent in full-term infants, neonatal sepsis can be a serious, life-threatening illness in newborns. Newborn infants who develop a temperature (generally over 38 C) must be presumed to have sepsis until proven otherwise. Localizing signs are generally not found in this age group, but should be carefully looked for. Such patients should be admitted on an urgent basis, and complete blood count, chest x-ray, and blood, urine, and cerebrospinal fluid cultures should be obtained. Broad-spectrum antibiotics should be started until the infant is afebrile and the culture results are negative. It is not appropriate to monitor newborns with fever at home, due to the risk of bacterial sepsis. Thus, acetaminophen (choice A) or arranging another visit in 2 days (choice B) are both unsuitable options in the management of this patient. While a CBC should be sent for this child, a normal blood count does not rule out bacterial sepsis (choice D). Neonatal sepsis requires treatment with parenteral antibiotics due to risk of septicemia or meningitis. Thus, oral amoxicillin (choice E) should not be started in this patient. The correct answer is D. This patient exhibits features of Turner syndrome, which has an abnormal karyotype (45,XO). The absence of one X chromosome leads to streak gonads and primary ovarian failure. Thus, there is absence of normal breast development during adolescence. Due to the presence of adrenal androgens, axillary and pubic hair development may occur. Short stature is an important sign of Turner syndrome and becomes more apparent when the peers undergo the growth spurt associated with normal puberty. In addition to the abnormal physical features described in the case vignette, Turner syndrome is associated with renal anomalies, cardiovascular anomalies, hypertension, hearing abnormalities, and, in the newborn period, edema of the hands and feet.

32.

The correct answer is D. This infant has ambiguous genitalia. The most common cause of ambiguous genitalia is female pseudohermaphroditism, and congenital adrenal hyperplasia (CAH) is the most common cause of female pseudohermaphroditism. 21-Hydroxylase enzyme deficiency is the most common enzyme deficiency and accounts for 90% of congenital adrenal hyperplasia. It is critical that the precise etiology be delineated so that any urgent metabolic abnormalities can be treated safely and quickly. In female pseudohermaphroditism, the gonadal tissue is represented by ovaries. The chromosomal analysis shows 46,XX. Overexposure to androgens in utero causes severe masculinization of the external genitalia. CAH patients display marked enlargement of the phallus, which excretes urine through a single urogenital sinus opening.. Many of these infants experience salt wasting caused by reduced aldosterone production. Salt wasting may cause a low plasma sodium with high renin and potassium concentrations. Death may occur in the neonatal period if the ensuing electrolyte abnormalities are unrecognized and untreated. Dehydration secondary to vomiting can lead to circulatory collapse. An arterial blood gas (choice A) is not helpful in the treatment of the underlying condition. Although the gas will reveal an acidosis, the source of the dehydration and lethargy is electrolyte abnormalities. Karyotype (choice B) is important to determine the exact chromosomal makeup of the infant. These results help in determining the sex of the infant for rearing, however, and are not available immediately. It is an important test, but not one that will help this lethargic, dehydrated infant in the intensive care unit.

29

USMLE Step 2 Assessment Exam

Pelvic ultrasound (choice C) can be used to determine the presence or absence of various intraabdominal organs or the presence of nonpalpable testes. It does not provide definitive answers, however, and will not help this dehydrated infant. In patients with CAH caused by 21-hydroxylase or 11beta-hydroxylase enzyme deficiency, the 17-alphahydroxyprogesterone serum level (choice E) is elevated. This piece of knowledge is important for determining the underlying etiology, but in the intensive care unit, the patient’s electrolytes are of higher priority. 33.

The correct answer is D. By making a family tree you can see that the described pattern of inheritance is consistent with X-linked dominant transmission. In this case the disease is manifested in female heterozygotes as well as carrier males (hemizygotes). Albinism (choice A) and congenital adrenal hyperplasia (choice B) are inherited in an autosomal recessive fashion. In autosomal recessive inheritance, there is a statistical probability that one of four children will be affected with the disease, two of four will be carriers, and one of four will be completely normal. There is no gender preference for the disease. Bruton agammaglobulinemia (choice C) and Duchenne muscular dystrophy (choice E) are inherited in an X-linked recessive fashion. The usual transmission is from heterozygous females to male offspring. Family history reveals that the disorder is only found in certain male relatives, and commonly in maternal uncles.

34.

The correct answer is A. Myelomeningocele is a neural tube defect and the most severe form of dysraphism involving the vertebral column. The etiology is unknown; however, it is thought that agents such as drugs, radiation, malnutrition, and genetics have a role in adversely affecting normal central nervous system development. After 1991, 50% of cases of neural tube defects in the United States are related to nutritional deficiency of folic acid and are thus preventable. Offspring in a family with one affected child are at increased risk of neural tube defect. The biochemical marker for neural tube defects is alpha-fetoprotein acetylcholinesterase (AFP). This substance is excreted from the fetus and leaks into the amniotic fluid when there is failure of the neural tube to close. Attention to nutrition in the prenatal period, with prenatal vitamins and folic acid, decreases the risk. The U.S. Public Health Service recommends intake of folic acid at a dosage of 0.4 mg/day for all women anticipating pregnancy. Since 1996, mandatory folic acid fortification of enriched cereal grain has been implemented in the United States.

30

Vitamin A (choice B) deficiency results in a skin eruption that consists of keratotic papules of varying size over the shoulders and extremities that may spread to involve most of the body surface except palms and soles (where there are no pilosebaceous follicles). On the face it resembles acne because of large comedones that develop. The whole skin displays dryness and fine scaling. Eye findings are prominent and often pathognomonic. They include night blindness (nyctalopia), an inability to see bright light, xerophthalmia, xerosis corneae, and keratomalacia. Vitamin A deficiency is a major cause of blindness in children in Third World countries. Vitamin B2 (choice C) deficiency manifests with the oral-ocular-genital syndrome. Angular cheilitis and cheilosis are prominent. The tongue is atrophic and magenta-colored. A seborrheic dermatitis–like eruption affects the eyebrows, glabella, and nasolabial folds of the face. Confluent dermatitis of the scrotum or vulva is present and extends onto the thighs. Photophobia and blepharitis angularis also occur. All the signs respond dramatically to 5 mg of riboflavin (vitamin B2) daily. Vitamin B6 (choice D) deficiency occurs in patients with uremia or cirrhosis, and with some medications. Skin changes are prominent and include a seborrheic dermatitis–like eruption, atrophic glossitis with ulceration, angular cheilitis, conjunctivitis, and intertrigo. The patients may be somnolent and confused, and may display signs of neuropathy. Vitamin K (choice E) deficiency occurs in patients with malabsorption syndromes or liver disease, or as a side effect of certain medications (coumarin, salicylates, cholestyramine). The result is a decrease in the vitamin K–dependent clotting factors II, VII, IX, and X. Purpura, hemorrhage, and ecchymoses occur in the skin. Treatment is with 5 to 10 mg/day of intramuscular vitamin K. In acute, life-threatening situations, fresh frozen plasma will provide the deficient clotting factors until their synthesis can be resumed. 35.

The correct answer is A. Friedrich ataxia is a hereditary metabolic degenerative disease caused by a repeat expansion of a gene that codes for the mitochondrial protein frataxin, resulting in an overload of iron in mitochondria. It is transmitted as an autosomal recessive or dominant trait. Ataxia usually appears before 10 years of age. Cardinal features include progressive limb and gait ataxia, dysarthria, loss of joint position and vibration senses, absent tendon reflexes in the legs, and extensor plantar responses. Patients have dysarthric speech and nystagmus. On physical examination, patients have ataxia and absent deep tendon reflexes. The lower

Pediatrics Answers and Explanations

extremities are usually more involved than the upper extremities, and there is also loss of vibration and position sense. Skeletal deformities, such as high arched foot (pes cavus), hammertoes, or kyphoscoliosis, may be present. Titubation is present when sitting or standing. The diagnosis is based on clinical signs and symptoms. There is no available treatment. This is a progressive disorder with significant morbidity. Loss of ambulation typically occurs 15 years after disease onset. More than 95% of patients are wheelchair-bound by age 45 years. Patients with Friedrich ataxia may develop hypertrophic cardiomyopathy, with progression to intractable congestive heart failure causing death. Refsum disease (choice B) is a neurocutaneous syndrome that is characterized by the accumulation of phytanic acid in plasma and tissues. Patients are unable to degrade phytanic acid because of a deficient activity of phytanoyl-CoA hydroxylase. Peripheral polyneuropathy, cerebellar ataxia, retinitis pigmentosa, and ichthyosis are the major clinical components. The symptoms evolve slowly and insidiously from childhood through adolescence and early adulthood. Spastic hemiplegia (choice C), a form of cerebral palsy, is a disorder of impaired motor functioning and posture with onset before or at birth or during the first year of life. It is a nonprogessive disorder and varies widely in its causes, manifestations, and prognosis. The most obvious manifestation is impaired ability of voluntary muscles. Spastic hemiplegia involves only one side of the body. Cognitive function may be spared, but seizures are common. Werdnig-Hoffman disease (choice D) is a degenerative disorder of motor neurons. The primary pathology is atrophy of anterior horn cells in the spinal cord and of motor nuclei in the brain stem, with secondary atrophy of motor nerve roots and of muscle. The disease is transmitted as an autosomal recessive trait. Onset is before 2 years of age and often begins in utero. Patients have generalized weakness and severe hypotonia of the proximal and distal limbs, intercostals, and bulbar muscles. Wilson disease (choice E) is an autosomal recessive degeneration of basal ganglia characterized by increased copper deposition in the brain, liver, kidney, and cornea, and low serum copper and ceruloplasmin levels. A family history is often present. Clinically, patients may have tremor, drooling, “fixed smile,” dysarthria, and choreoathetosis. Children may have hepatomegaly. Almost all patients with neurologic disease have Kayser-Fleischer rings, i.e., a yellow-brown deposit at the limbus of the cornea. The diagnosis of Wilson disease is confirmed by low serum copper and ceruloplasmin levels, increased urine copper concentrations after the administration of penicillamine, and a rise in the level of copper in the liver.

36.

The correct answer is E. Lesch-Nyhan syndrome is an X-linked disorder of purine metabolism deficiency of hypoxanthine-guanine phosphoribosyl transferase. It is, therefore, seen almost exclusively in boys. This metabolic deficiency leads to an excess of uric acid. Infants with Lesch-Nyhan syndrome have no apparent neurologic dysfunction until a delay in motor development occurs in the first few months. This is usually the first abnormality noted. In addition, patients have selfdestructive behavior patterns. On physical examination, there is evidence of self-mutilation, choreoathetosis, spasticity, psychomotor retardation, gouty arthritis, and renal calculi. Initially, uric acid crystalluria and microhematuria may present as “orange sand” noticed in the diapers. Plain abdominal radiographs (choice D) will not detect calculi composed of uric acid, oxypurine metabolites, or allopurinol because they are radiolucent. However, they are easily seen by ultrasonography. The diagnosis may be made by the presence of dystonia and self-mutilation. The definitive diagnosis is made by analysis of the hypoxanthine-guanine phosphoribosyl transferase enzyme. Treatment consists of allopurinol for renal complications. Behavior modification, restraints, and removal of teeth are needed to reduce anxiety and stabilize mood. Cerebrospinal fluid analysis (choice A) does not reveal any abnormality in patients with Lesch-Nyhan syndrome. Computerized tomography of the brain (choice B) and magnetic resonance imaging of the brain (choice C) generally do not reveal any structural abnormalities or signal changes in the gray or white matter. Even if there is mild loss of brain volume, it is most often too small to be detected in routine imaging studies. Neuroimaging of the spinal cord may reveal early degenerative joint disease with possible damage to the spinal cord or nerve roots.

37.

The correct answer is C. This patient has fulminant hepatic failure associated with hepatolenticular degeneration. Liver biopsy is the criterion standard for diagnosis. Hepatolenticular degeneration (Wilson disease) is an autosomal recessive degeneration of basal ganglia characterized by increased copper deposition in the brain, liver, kidney, and cornea, and low serum copper and ceruloplasmin levels. A family history is often present, and screening asymptomatic family members identifies 25% of cases. Initial presentation of Wilson disease occurs between adolescence and 40 years of age. The clinical presentation varies, although it has been noted that hepatic failure is more common in children whereas psychiatric symptoms are more common in adults. Clinically, patients may have tremor, drooling, “fixed smile,” dysarthria, and choreoathetosis. Children

31

USMLE Step 2 Assessment Exam

may have hepatomegaly. Almost all patients with neurologic disease have Kayser-Fleischer rings, i.e., a yellow-brown deposit at the limbus of the cornea. The diagnosis of Wilson disease is confirmed by low serum copper and ceruloplasmin levels, increased urine copper concentrations after the administration of penicillamine, and a rise in the level of copper in the liver. Analysis of 24-hour urine collection (choice A) will reveal increased urinary copper excretion in excess of 100 mg/dL. Further increase in copper excretion after penicillamine administration is used to confirm the diagnosis. Nevertheless, a liver biopsy remains the gold standard for diagnosis. Computerized tomography of the brain (choice B) is used to assess the presence of hypodense regions in the basal ganglia of patients with Wilson disease. Other possible findings include ventricular dilatation, brainstem atrophy, and posterior fossa atrophy. Skeletal radiologic survey (choice D) is not recommended as part of the workup for Wilson disease in children because the musculoskeletal abnormalities do not develop until adulthood. Slit-lamp examination of the eye (choice E) is necessary for confirming the presence of Kayser-Fleischer rings, copper granules in the stromal layer of the eye. They are always present with neurologic disease, and may be present in hepatic disease as well. Kayser-Fleischer rings fade and disappear with appropriate therapy. 38.

32

The correct answer is D. Although uncommon, primary osteosarcoma, a malignant bone tumor, occurs most frequently in adolescents and teens. The distal femur is the most common location for osteosarcoma, but it may present almost anywhere in the appendicular skeleton. In the past, treatment for osteosarcoma was limited to amputation with a very low survival rate; however, with chemotherapy and limb salvage surgery, the mortality of osteosarcoma has been decreased significantly. A benign bone cyst (aneurysmal or unicameral) (choice A) is generally not associated with thigh swelling or pain. Radiographically, cysts are radiolucent and do not appear to form bone. There is no mention of a distal femur fracture in the patient’s history, and although callous formation may be difficult to interpret on radiographic examination, the patient’s history should give one that diagnosis; thus, choice B is incorrect. The patient does not have a history of fever, chills, or night sweats and has an enlarging mass in her distal femur; this is not suggestive of osteomyelitis (choice C).

Paget disease is a pathologic condition that results in abnormal bone turnover and is not associated with enlarging, bone-forming masses (choice E). 39.

The correct answer is A. This patient has the classic clinical presentation of Rett syndrome. Rett syndrome (pervasive developmental disorder) is a neurodegenerative disorder that affects only females. The onset usually occurs at 1 year of age with loss of developmental milestones and acquired microcephaly. On physical examination, there is a loss of purposeful movement, social withdrawal, stereotypical hand movements (hand wringing), and acquired microcephaly. The patient may have sighing, intermittent apnea, and autistic behavior. Currently, diagnosis of Rett syndrome is made if the patient meets defined clinical criteria. The diagnosis is supported by a positive mutational analysis of MECP2 (methyl-CpG binding protein-2). The gene is located on chromosome X. However, as many as 20% of females meeting the full clinical criteria for Rett syndrome may have no identified mutation. It is considered to be fatal in males. Because no cure exists, treatment is palliative and supportive. A multidisciplinary approach to care for persons with Rett syndrome is recommended. If the patient has seizures, anticonvulsants should be administered. These patients may have generalized tonic-clonic seizures. Death occurs during adolescence or the third decade, and may be caused suddenly from a cardiac arrhythmia. NF1 (choice B) mutations or deletions are responsible for the development of type 1 neurofibromatosis. The gene is located on chromosome 17. These patients are at increased risk of developing nervous system neoplasms, including plexiform neurofibromas, optic gliomas, ependymomas, meningiomas, astrocytomas, and pheochromocytomas. Neurofibromas may undergo secondary malignant degeneration and become sarcomas. PTEN (choice C) is a tumor suppressor gene located on chromosome 10q23. Mutation of this gene results in the multiple hamartoma syndrome (Cowden disease), characterized by the development of trichilemmomas, oral mucous papillomas, acral keratoses, breast cancer in women, and medullary carcinoma of the thyroid in both men and women. SPINK5 (choice D) is a serine protease inhibitor of Kazal type responsible for the genetic defect that results in Netherton syndrome, a rare autosomal recessive genodermatosis of cause characterized by erythroderma, trichorrhexis invaginata (bamboo hair), ichthyosis linearis circumflexa, atopic diathesis, and failure to thrive. TSC1 (choice E) is the gene responsible for tuberous sclerosis. It is located on chromosomes 9 and 16.

Pediatrics Answers and Explanations

Tuberous sclerosis is a neurocutaneous syndrome characterized by mental retardation, facial fibroangiomas, hypopigmented spots of the skin, and epilepsy. 40.

41.

The correct answer is B. Most single congenital malformations follow the pattern of multigenic, or nonMendelian, inheritance. Both genetic and environmental factors are thought to be involved in this form of inheritance. Most common congenital malformations fall into this category. The recurrence risk of the same congenital malformation occurring in a second offspring is generally in the range of 4 to 6%. It is worth noting that the concordance rate among monozygotic twins is well below 100%, indicating the role of environmental factors. By its very nature, multigenic inheritance does not involve specific genes or cytogenetic markers. Thus, chromosomal studies (choice A) are generally not helpful in most isolated common congenital malformations such as clubfoot, pyloric stenosis, or cleft lip. The risk of recurrence is higher for conditions that clearly follow Mendelian inheritance. These are singlegene defects such as sickle-cell anemia or cystic fibrosis. In these recessively inherited diseases, the recurrence risk in the second child is 25% (choice C). As compared with the normal population, the risk of recurrence is increased about 20 to 50 times in congenital malformations that follow the pattern of multigenic inheritance (choice D). Although the incidence of certain chromosomal defects (such as trisomy) increases with higher maternal age, the same is not true of diseases such as cleft palate, pyloric stenosis, or clubfoot that follow multigenic inheritance (choice E). The correct answer is E. This child demonstrates signs of impending respiratory failure and should be immediately intubated in the emergency room. Patients in status asthmaticus should be evaluated for altered mental status, retention of carbon dioxide, and development of acidosis. These patients are at risk of dying due to respiratory failure unless the condition is recognized in time. Children with severe asthma may have a silent chest with no wheezing due to poor air exchange. They are unable to talk in full sentences due to hypoxia, and they need to breathe in while talking. Children presenting with status asthmaticus may be dehydrated due to tachypnea and decreased intake of fluids. Thus, although intravenous fluids (choice A) are required in the management of this child, they should be provided after the breathing has been stabilized. Methylprednisolone (choice B) or other steroids are essential in treating status asthmaticus. However, the

onset of action is not immediate and thus other steps must be taken to improve ventilation. Sodium bicarbonate (choice C) is sometimes used for severe acidosis, especially where ongoing bicarbonate loss is suspected. It should not be used in patients with inadequate ventilation. It is not adequate to admit the child for observation alone (choice D), given the extreme respiratory distress. 42.

The correct answer is C. This patient has the typical presentation of Duchenne muscular dystrophy. The diagnosis is confirmed by muscle biopsy. Duchenne muscular dystrophy is the most common hereditary neuromuscular disease. It is caused by a mutation in the dystrophin gene at Xp2.1. It is transmitted as an Xlinked recessive disease, with approximately 30% new mutations. Clinically, poor head control may be one of the first signs of the disease. There could be mild delay in early gross motor skills. On physical examination, patients will have pseudohypertrophy of the calves. Gowers sign (pushing the knees to stand up) is seen by 3 years of age. Hip girdle weakness and a Trendelenburg sign (at 5 to 6 years of age) develop. Patients eventually lose ambulation capabilities. Patients have a poor cough, and pharyngeal weakness. Cardiomyopathy also develops. Diagnosis is made by muscle biopsy demonstrating necrosis, fat cells, and fibrous tissue. Creatine kinase is greatly elevated. Becker muscular dystrophy is also caused by a deficiency of dystrophin, but the age of onset is later and the course of the disease is slower. Pseudohypertrophy is prominent, and pes cavus deformities are present. Cardiac and nervous system involvement is unusual. An electrocardiogram (choice A) is helpful in uncovering sinus arrhythmia with deep Q waves and elevated right precordial R waves, but has no value in confirming the diagnosis of Duchenne muscular dystrophy. Electromyography (choice B) is not diagnostic but it narrows the differential diagnosis, effectively excluding primarily neurogenic processes. Radiographs of the spine (choice D) are important for screening and evaluating the degree of scoliosis in patients who are wheelchair-bound, but has no value in confirming the diagnosis of Duchenne muscular dystrophy. Serum creatinine phosphokinase (choice E) is always increased in patients with Duchenne muscular dystrophy or Becker muscular dystrophy, up to 50 to 100 times the reference range. If a child has very elevated creatinine phosphokinase and proximal muscle weakness, further testing to confirm the diagnosis is indicated.

33

USMLE Step 2 Assessment Exam

43.

The correct answer is D. The clinical presentation of cystic fibrosis is variable, with a significant number of infants manifesting pulmonary symptoms. These can be chronic cough, recurrent pneumonia or bronchiolitis, wheezing, atelectasis, or hemoptysis. Infants who present with a picture of acute bronchiolitis repeatedly or outside the winter season should be suspected to have an ongoing problem such as asthma, cystic fibrosis, foreign body, or congenital malformation. In this case, hyperexpanded lungs, failure to thrive, and presence of loose, foul-smelling, greasy stools (indicative of exocrine pancreatic involvement) should lead to referral for performing a sweat chloride test. Elevated sweat chloride (>60 mEq/L) in the presence of suggestive symptoms is diagnostic of cystic fibrosis. Children with persistent or recurrent pulmonary symptoms should be evaluated for possible foreign-body aspiration by bronchoscopy (choice A). The presence of failure to thrive and intestinal symptoms make this a less likely diagnosis. Chest x-ray (choice B) is indicated in this case and would reveal hyperinflation and patchy atelectasis. These findings are not specific for cystic fibrosis. Certain patients with protracted pulmonary symptoms and diffuse lung changes on x-ray require a lung biopsy (choice C) to establish a diagnosis. Sweat chloride testing is a more definitive and less invasive method of diagnosing cystic fibrosis, compared with lung biopsy. Children with malabsorption may require GI endoscopy (choice E) to obtain a mucosal biopsy for diagnosis. However, prominent respiratory symptoms in this case indicate the presence of a systemic disorder, and sweat chloride testing is the appropriate diagnostic test.

44.

The correct answer is C. This girl has pediculosis capitis (head lice). Microscopic examination of the snipped hair would help identify nits attached to hair shafts. Itching is the principal manifestation of this infestation and the lice should not be overlooked because of impetigo of the scalp or suboccipital lymphadenopathy, both secondary to scratching. Dermatitis of the neck, shoulders, and urticaria may also occur as a hypersensitivity reaction to the pediculus. It presents as red, scaly, lichenified plaques on the posterior neck and shoulders. Head lice can survive up to 2 days, or longer, off the scalp, so shared hats, headsets, combs, brushes, bedding, clothing or upholstery easily transmits pediculosis. Nits are mainly found on the hairs in the postauricular and occipital scalp as small, whitish, 0.5-mm nodules. They are easily spotted in dark-haired individuals but may be camouflaged well in blondes. Nits should not be mistaken for epidermal

34

scale or hair casts, both of which are easily slid down the hair shaft, whereas nits are firmly attached and cannot be easily moved. In addition, nits fluoresce under Wood’s light and are easily viewed on snipped hairs under the microscope. Sabouraud’s dextrose agar (choice A) is the most commonly used medium in medical mycology and serves as the basis for most of the morphologic descriptions of these fungi. Cultures should be maintained at room temperature (26 C) for up to 4 weeks before they are discarded as showing no growth. The hair penetration test (choice B) is used to identify certain fungal cultures in vitro. Species of Trichophyton will penetrate a hair shaft placed in the medium, whereas those of Microsporum, for example, will not. Potassium hydroxide hair preparation (choice D) is used to diagnose tinea capitis. The hair shaft is immersed in a 20% potassium hydroxide solution for several hours, and then examined under the microscope. This helps dissolve the hair keratins and permits visualization of the fungi. The pull test (choice E) is utilized in the diagnosis and assessment of alopecia areata, a disorder in which round or oval patches of hair precipitously fall out leaving a smooth but otherwise normal skin surface with visible hair follicle orifices but no hair shafts. The pull test is performed at the periphery of such patches by firmly grasping approximately 50 hairs close to the scalp and firmly pulling. If more than 5 hairs are liberated from the scalp by this maneuver, the inflammatory process leading to hair loss is still active in that area of the scalp. 45.

The correct answer is E. Mothers are advised to place their infants “back to sleep.” Studies looking at SIDS rates during the introduction of such “Back to Sleep” campaigns have found a decline in occurrences of SIDS by nearly 40%, though a causal relationship has not yet been proven. To the chagrin of their parents, many infants refuse to sleep in one position. In general, any comfortable position (choice A) other than prone significantly reduces the infant’s chances of SIDS. The supine sleeping position followed by sleeping on either side (choices B and D) is safer than the prone position. The prone sleeping position (choice C) has been correlated with an increased risk for SIDS in several retrospective studies. Although the exact cause of increased risk with sleeping in the prone position has not been proven, this position is hypothesized to increase the risk for smothering and potentiating apneic episodes.

Pediatrics Answers and Explanations

46.

47.

The correct answer is C. This patient meets the criteria for Kawasaki syndrome, the most common vasculitis among children age 5 years and younger. Patients must have an otherwise unexplained fever, together with four of the five following symptoms: conjunctivitis, mucous membrane changes, cervical lymphadenopathy, rash, and swelling or desquamation of the hands or feet. The disease is most common in children of Japanese descent and often can be mistaken for scarlet fever or a nonspecific viral syndrome. Prompt diagnosis and treatment with aspirin and intravenous immunoglobulin is essential. Up to a quarter of untreated patients develop dilated coronary arteries that are prone to stenosis and thrombosis. Acetaminophen (choice A) can mask the fevers and treat some of the associated symptoms, such as arthritis. Aspirin (choice B) has been found to be much more effective, however, because of its anti-inflammatory effects and its antithrombotic effects. Aspirin alone, however, does not reduce the incidence of coronary artery aneurysm, which intravenous immunoglobulin (IVIG) does. As such, the first-line treatment for Kawasaki syndrome is combination therapy with IVIG and aspirin. Plasmapheresis (choice D) and prednisone (choice E) generally are not used for the treatment of Kawasaki syndrome, though both may be used in rare instances as salvage therapy. Steroids, effective in most forms of vasculitis, have been associated with increased occurrence of coronary artery aneurysms. Plasmapheresis, on the other hand, has mainly been limited by technical considerations and the availability of a more tested medical treatment. The correct answer is A. This patient has Turner syndrome, usually caused by absence or abnormality of an X chromosome, resulting at least functionally in a patient with a genotype of 45,XO. In addition to primary amenorrhea, sterility, classic physical findings of micrognathia, and a “shield chest,” these patients are at an increased risk for coarctation of the aorta. Up to 20% of patients with Turner syndrome have this condition, which can cause primarily upper extremity hypertension. Essential hypertension (choice B) is the most common cause of hypertension, accounting for 90 to 95% of cases. It is rare, however, to have essential hypertension in such a young patient. The associated physical findings point toward Turner syndrome, commonly associated with coarctation of the aorta, a well-described cause of secondary hypertension. Ovarian failure (choice C) is part of Turner syndrome. Strands of nonfunctional connective tissue replace the

ovaries. Ovarian failure, however, does not cause hypertension. Rather the associated cases of coarctation may result in hypertension. Renal artery stenosis (choice D) and renal failure (choice E) are not commonly associated with this condition and are unlikely to be a cause of this patient’s symptoms. 48.

The correct answer is D. Spina bifida occulta is the most benign form of dysraphism. There is a defect of the closure of the posterior vertebral arches and laminae, usually at L5 and S1. The meninges do not herniate through the bony defect. The spinal defect is covered by skin rendering the underlying neurologic defect occult or hidden. These children do not have associated hydrocephalus or Chiari II malformations. Orthopedic malformations include extremity asymmetry and foot deformities. However, most children will be asymptomatic. Typically, a dermal sinus or patch of hair may be present over the defect. A spinal roentgenogram shows a defect in the closure of the vertebral arches. Lamina defects, hemivertebrae, scoliosis, widening of interpedicular spaces, and butterfly vertebrae may all be seen on x-ray. Surgical repair of these lesions is most effectively performed in a prophylactic fashion. Ash leaf macules (choice A) are hypopigmented macules present on the skin of patients with tuberous sclerosis. They are usually the first visible cutaneous marker of this congenital disorder and may be present as a single or multiple lesions. Tuberous sclerosis patients may also develop angiofibromas of the face (adenoma sebaceum) and periungually (Koenen tumors). Calcifications may be present in the central nervous system, leading to a seizure disorder that usually begins in teenage or adolescent years. Often, there is mental retardation of variable degree. A Becker nevus (choice B) is visible as a tan or brown macule present at birth, most commonly on the shoulder area. It is characterized by flattened and broad rete ridges that have an increased number of melanocytes present at the dermoepidermal junction. About 15% will have an associated intradermal melanocytic nevus. At puberty, thick, dark terminal hairs may develop within the lesion. There are no associated central nervous system abnormalities. Mongolian spots (choice C) are flat blue or slate-gray lesions with well-defined margins, present at birth. Most commonly, they are located in the lumbosacral area, although they may appear on other parts of the skin. Mongolian spots are very common in dark-skinned races, especially Asian, where they are seen in up to 90% of newborns. A total of 5 to 10% of Caucasian newborns

35

USMLE Step 2 Assessment Exam

will have one at birth. Most of these lesions resolve in the first few years of life, but in a small percentage of cases they will persist into adulthood. Histopathologically, melanin-containing melanocytes are seen in the dermis. Mongolian spots should be differentiated from the bruises of child abuse, which have variation of color within them and gradually fade into the surrounding skin. Salmon patch (choice E), or nevus flammeus, is a common capillary malformation of the skin, present in at least 15% of newborns. It may persist indefinitely in 5% of newborns and grow in proportion to the growth of the affected body part. A pink-red macule is seen at birth on the glabella or on one upper eyelid, where they appear in about 15% of infants. In this location, nevus flammeus has a tendency to fade and disappear some time during childhood (as opposed to the “stork bite” lesion of the nuchal region, where it is somewhat more common and tends to persist indefinitely). 49.

36

The correct answer is C. The key to the question is the patient’s age. Legg-Calve-Perthes disease is a noninflammatory deformity of the proximal femur caused by a vascular insult, leading to osteonecrosis of the proximal femoral epiphysis. It is most commonly seen in boys aged 4 to 8 years with delayed skeletal maturity, but should be considered in any limping child 4 to 12 years of age. Patients may be pain-free or note knee, groin, or back pain. Decreased range of motion of the affected hip and a Trendelenburg gait are also common findings. This patient is too old for developmental dysplasia of the hip (choice A) to be the most likely diagnosis because it is usually recognized within the first 2 years of life. Although juvenile rheumatoid disease (choice B) is a possibility, it too is much less likely than Legg-CalvePerthes disease. Juvenile rheumatoid arthritis usually affects patients older than the boy in this scenario (greater than 8 years of age) and is much more common in girls. The patient does not appear septic or have any history of fever, chills, or severe pain with hip range of motion. Thus, septic arthritis (choice D) is highly unlikely. Slipped capital femoral epiphysis (choice E) usually occurs in adolescents and is often associated with obesity or endocrine disorders.

50.

The correct answer is B. Administration of hepatitis B vaccine and hepatitis B immune globulin immediately after birth appears to give the infant an initially high level of antibodies against the virus; and the vaccine, when all three doses of the vaccine have been administered, affords long-term protection. This is now considered standard care for an infant with a mother who is hepatitis B surface antigen–positive. Administration of hepatitis B vaccine only (choice A), while definitely appropriate to administer to this child, will not result in an initially high level of protective antibodies in the newborn. Administration of hepatitis B immune globulin only (choice C) affords only transient protection to the newborn and still exposes to the child to further infection later in life. Explaining to the mother that no intervention is indicated (choice D) is untrue; furthermore, it could possibly prevent the administration of medications that could possibly protect the newborn from infection. It has also been shown that women who have surface antigenemia with a high level of infectivity have a >90% transmission rate from mother to fetus, as is the case in this scenario. Waiting to repeat a serologic workup before treatment of the infant (choice E) denies the newborn the highest level of protection for postexposure prophylaxis. Current recommendations state that the immune globulin should be administered immediately after birth and the first dose of the vaccine within the first 12 hours of birth.

BL3162A Printed in USA © 2003

kaplanmedical.com

View more...

Comments

Copyright ©2017 KUPDF Inc.
SUPPORT KUPDF